128 Fundamentals of Mathematics: Part 10

Exercise 7.5.24

(Solution on p. 470.)

15%

Exercise 7.5.25

43%

Exercise 7.5.26

(Solution on p. 470.)

16.2%

Exercise 7.5.27

53.8%

Exercise 7.5.28

(Solution on p. 470.)

5.05%

Exercise 7.5.29

6.11%

Exercise 7.5.30

(Solution on p. 470.)

0.78%

Exercise 7.5.31

0.88%

Exercise 7.5.32

(Solution on p. 470.)

0.09%

Exercise 7.5.33

0.001%

For the following 14 problems, convert each fraction to a percent.

Exercise 7.5.34

(Solution on p. 470.)

1

5

Exercise 7.5.35

3

5

Exercise 7.5.36

(Solution on p. 470.)

5

8

Exercise 7.5.37

1

16

Exercise 7.5.38

(Solution on p. 470.)

7

25

Exercise 7.5.39

16

45

Exercise 7.5.40

(Solution on p. 470.)

27

55

Exercise 7.5.41

15

8

Exercise 7.5.42

(Solution on p. 470.)

41

25

Exercise 7.5.43

6 45

Exercise 7.5.44

(Solution on p. 470.)

9 9

20

Available for free at Connexions <http://cnx.org/content/col10615/1.4>

443

Exercise 7.5.45

1

200

Exercise 7.5.46

(Solution on p. 470.)

6

11

Exercise 7.5.47

35

27

For the following 14 problems, convert each percent to a fraction.

Exercise 7.5.48

(Solution on p. 470.)

80%

Exercise 7.5.49

60%

Exercise 7.5.50

(Solution on p. 470.)

25%

Exercise 7.5.51

75%

Exercise 7.5.52

(Solution on p. 470.)

65%

Exercise 7.5.53

18%

Exercise 7.5.54

(Solution on p. 470.)

12.5%

Exercise 7.5.55

37.5%

Exercise 7.5.56

(Solution on p. 470.)

512.5%

Exercise 7.5.57

937.5%

Exercise 7.5.58

(Solution on p. 470.)

_

9. 9 %

Exercise 7.5.59

_

55. 5 %

Exercise 7.5.60

(Solution on p. 470.)

_

22. 2 %

Exercise 7.5.61

_

63. 6 %

7.5.4.1 Exercises for Review

Exercise 7.5.62

(Solution on p. 470.)

(Section 4.6) Find the quotient. 40

54 ÷ 8 7

21 .

Exercise 7.5.63

(Section 4.7) 3 of what number is 22?

8

3

Exercise 7.5.64

(Solution on p. 470.)

(Section 5.3) Find the value of 28

15 + 7

10 − 5

12 .

Available for free at Connexions <http://cnx.org/content/col10615/1.4>

444

CHAPTER 7. RATIOS AND RATES

Exercise 7.5.65

(Section 6.4) Round 6.99997 to the nearest ten thousandths.

Exercise 7.5.66

(Solution on p. 470.)

(Section 7.4) On a map, 3 inches represent 40 miles. How many inches represent 480 miles?

7.6 Fractions of One Percent6

7.6.1 Section Overview

• Conversions Involving Fractions of One Percent

• Conversions Involving Nonterminating Fractions

7.6.2 Conversions Involving Fractions of One Percent

Percents such as 1%, 3 %, 5 %, and 7

2

5

8

11 %, where 1% has not been attained, are fractions of 1%. This implies that

1 % = 1 of 1%

2

2

3 % = 3 of 1%

5

5

5 % = 5 of 1%

8

8

7

11 % = 7

11 of 1%

Since “percent” means “for each hundred,” and “of” means “times,” we have 1 % = 1 of 1% = 1 · 1

2

2

2

100 = 1

200

3 % = 3 of 1% = 3 · 1

5

5

5

100 = 3

500

5 % = 5 of 1% = 5 · 1

8

8

8

100 = 5

800

7

11 % = 7

11 of 1% = 7

11 · 1

100 =

7

1100

7.6.2.1 Sample Set A

Example 7.31

Convert 2% to a fraction.

3

1

2 % = 2 of 1% = )2 · 1

3

3

3

)100

50

=

1·1

3·50

=

1

150

Example 7.32

Convert 5 % to a decimal.

8

6This content is available online at <http://cnx.org/content/m34997/1.2/>.

Available for free at Connexions <http://cnx.org/content/col10615/1.4>

445

5 % = 5 of 1% =

5 · 1

8

8

8

100

=

0.625 · 0.01

=

0.00625

7.6.2.2 Practice Set A

Exercise 7.6.1

(Solution on p. 470.)

Convert 1 % to a fraction.

4

Exercise 7.6.2

(Solution on p. 471.)

Convert 3 % to a fraction.

8

Exercise 7.6.3

(Solution on p. 471.)

Convert 31 % to a fraction.

3

7.6.3 Conversions Involving Nonterminating Fractions

We must be careful when changing a fraction of 1% to a decimal. The number 2, as we know, has a 3

nonterminating decimal representation. Therefore, it cannot be expressed exactly as a decimal.

When converting nonterminating fractions of 1% to decimals, it is customary to express the fraction as a rounded decimal with at least three decimal places.

Converting a Nonterminating Fraction to a Decimal

To convert a nonterminating fraction of 1% to a decimal:

1. Convert the fraction as a rounded decimal.

2. Move the decimal point two digits to the left and remove the percent sign.

7.6.3.1 Sample Set B

Example 7.33

Convert 2 % to a three-place decimal.

3

1. Convert 2 to a decimal.

3

Since we wish the resulting decimal to have three decimal digits, and removing the percent sign will account for two of them, we need to round 2 to one place (2 + 1 = 3).

3

2 % = 0.7% to one decimal place. 2 = 0.6666 . . .

3

3

2. Move the decimal point two digits to the left and remove the % sign. We’ll need to add zeros to locate the decimal point in the correct location.

2 % = 0.007 to 3 decimal places

3

Example 7.34

Convert 5 411% to a four-place decimal.

1. Since we wish the resulting decimal to have four decimal places, and removing the percent sign will account for two, we to round 411 to two places.

5 4

11 % = 5.36% to two decimal places.

4

11 = 0.3636 . . .

2. Move the decimal point two places to the left and drop the percent sign. 5 411% = 0.0536 to four decimal places.

Available for free at Connexions <http://cnx.org/content/col10615/1.4>

446

CHAPTER 7. RATIOS AND RATES

Example 7.35

Convert 285% to a decimal rounded to ten thousandths.

9

1. Since we wish the resulting decimal to be rounded to ten thousandths (four decimal places), and removing the percent sign will account for two, we need to round 5 to two places.

9

285% = 28.56% to two decimal places. 5 = 0.5555 . . .

9

9

2. Move the decimal point to the left two places and drop the percent sign.

285% = 0.2856 correct to ten thousandths.

9

7.6.3.2 Practice Set B

Exercise 7.6.4

(Solution on p. 471.)

Convert 7% to a three-place decimal.

9

Exercise 7.6.5

(Solution on p. 471.)

Convert 51 5 % to a decimal rounded to ten thousandths.

11

7.6.4 Exercises

Make the conversions as indicated.

Exercise 7.6.6

(Solution on p. 471.)

Convert 3 % to a fraction.

4

Exercise 7.6.7

Convert 5% to a fraction.

6

Exercise 7.6.8

(Solution on p. 471.)

Convert 1% to a fraction.

9

Exercise 7.6.9

Convert 15

19 % to a fraction.

Exercise 7.6.10

(Solution on p. 471.)

Convert 5 % to a fraction.

4

Exercise 7.6.11

Convert 7% to a fraction.

3

Exercise 7.6.12

(Solution on p. 471.)

Convert 16% to a fraction.

7

Exercise 7.6.13

Convert 2 516% to a fraction.

Exercise 7.6.14

(Solution on p. 471.)

Convert 251% to a fraction.

4

Exercise 7.6.15

Convert 501% to a fraction.

2

Exercise 7.6.16

(Solution on p. 471.)

Convert 723% to a fraction.

5

Exercise 7.6.17

Convert 991% to a fraction.

8

Exercise 7.6.18

(Solution on p. 471.)

Convert 1362% to a fraction.

3

Available for free at Connexions <http://cnx.org/content/col10615/1.4>

447

Exercise 7.6.19

Convert 5213% to a fraction.

4

Exercise 7.6.20

(Solution on p. 471.)

Convert 101% to a decimal.

5

Exercise 7.6.21

Convert 123% to a decimal.

4

Exercise 7.6.22

(Solution on p. 471.)

Convert 37% to a decimal.

8

Exercise 7.6.23

Convert 7 116% to a decimal.

Exercise 7.6.24

(Solution on p. 471.)

Convert 3% to a three-place decimal.

7

Exercise 7.6.25

Convert 1% to a three-place decimal.

9

Exercise 7.6.26

(Solution on p. 471.)

Convert 6 311% to a four-place decimal.

Exercise 7.6.27

Convert 92% to a four-place decimal.

7

Exercise 7.6.28

(Solution on p. 471.)

Convert 24 521% to a three-place decimal.

Exercise 7.6.29

Convert 45 827% to a three-place decimal.

Exercise 7.6.30

(Solution on p. 471.)

Convert 1116

17 % to a four-place decimal.

Exercise 7.6.31

Convert 51% to a three-place decimal.

7

7.6.4.1 Exercises for Review

Exercise 7.6.32

(Solution on p. 471.)

(Section 3.2) Write 8 · 8 · 8 · 8 · 8 using exponents.

Exercise 7.6.33

(Section 4.3) Convert 47 to an improper fraction.

8

Exercise 7.6.34

(Solution on p. 471.)

(Section 5.4) Find the sum. 710 + 221 + 1.

7

Exercise 7.6.35

(Section 6.6) Find the product. (4.21) (0.006).

Exercise 7.6.36

(Solution on p. 471.)

(Section 7.5) Convert 8.062 to a percent.

Available for free at Connexions <http://cnx.org/content/col10615/1.4>

448

CHAPTER 7. RATIOS AND RATES

7.7 Applications of Percents7

7.7.1 Section Overview

• Base, Percent, and Percentage

• Finding the Percentage

• Finding the Percent

• Finding the Base

7.7.2 Base, Percent, and Percentage

There are three basic types of percent problems. Each type involves a base, a percent, and a percentage, and when they are translated from words to mathematical symbols each becomes a multiplication statement.

Examples of these types of problems are the following:

1. What number is 30% of 50? (Missing product statement.)

2. 15 is what percent of 50? (Missing factor statement.)

3. 15 is 30% of what number? (Missing factor statement.)

In problem 1 (list, p. 448), the product is missing. To solve the problem, we represent the missing product with P .

P = 30% · 50

Percentage

The missing product P is called the percentage. Percentage means part, or portion. In P = 30% · 50, P

represents a particular part of 50.

In problem 2 (list, p. 448), one of the factors is missing. Here we represent the missing factor with Q.

15 = Q · 50

Percent

The missing factor is the percent. Percent, we know, means per 100, or part of 100. In 15 = Q · 50, Q

indicates what part of 50 is being taken or considered. Specically, 15 = Q · 50 means that if 50 was to be divided into 100 equal parts, then Q indicates 15 are being considered.

In problem 3 (list, p. 448), one of the factors is missing. Represent the missing factor with B.

15 = 30% · B

Base

The missing factor is the base. Some meanings of base are a source of supply, or a starting place. In 15 = 30% · B, B indicates the amount of supply. Specically, 15 = 30% · B indicates that 15 represents 30% of the total supply.

Each of these three types of problems is of the form

(percentage) = (percent) · (base)

We can determine any one of the three values given the other two using the methods discussed in Section 4.7.

7This content is available online at <http://cnx.org/content/m35007/1.2/>.

Available for free at Connexions <http://cnx.org/content/col10615/1.4>

449

7.7.3 Finding the Percentage

7.7.3.1 Sample Set A

Example 7.36

What number is

30%

of

50?

Missing product statement.

|

{z

}

(percentage)

= (percent) · (base)

P

=

30%

·

50

Convert 30% to a decimal.

P

=

.30

·

50

Multiply.

P

=

15

Thus, 15 is 30% of 50.

Do Section 7.7.3.2 (Practice Set A), Exercise 7.7.1.

Example 7.37

What number is

36%

of

95?

Missing product statement.

|

{z

}

(percentage)

= (percent) · (base)

P

=

36%

·

95

Convert 36% to a decimal.

P

=

.36

·

95

Multiply

P

=

34.2

Thus, 34.2 is 36% of 95.

Do Section 7.7.3.2 (Practice Set A), Exercise 7.7.1.

Example 7.38

A salesperson, who gets a commission of 12% of each sale she makes, makes a sale of $8,400.00.

How much is her commission?

We need to determine what part of $8,400.00 is to be taken. What part indicates percentage.

What number is

12%

of 8, 400.00? Missing product statement.

|

{z

}

(percentage)

= (percent) ·

(base)

P

=

12%

·

8, 400.00

Convert to decimals.

P

=

.12

·

8, 400.00

Multiply.

P

=

1008.00

Thus, the salesperson’s commission is $1,008.00.

Do Section 7.7.3.2 (Practice Set A), Exercise 7.7.2.

Example 7.39

A girl, by practicing typing on her home computer, has been able to increase her typing speed by 110%. If she originally typed 16 words per minute, by how many words per minute was she able to increase her speed?

We need to determine what part of 16 has been taken. What part indicates percentage.

Available for free at Connexions <http://cnx.org/content/col10615/1.4>

450

CHAPTER 7. RATIOS AND RATES

What number is

110%

of

16?

Missing product statement.

|

{z

}

(percentage)

= (percent) · (base)

P

=

110%

·

16

Convert to decimals.

P

=

1.10

·

16

Multiply.

P

=

17.6

Thus, the girl has increased her typing speed by 17.6 words per minute. Her new speed is 16 + 17.6 =

33.6 words per minute.

Do Section 7.7.3.2 (Practice Set A), Exercise 7.7.3.

Example 7.40

A student who makes $125 a month working part-time receives a 4% salary raise. What is the student’s new monthly salary?

With a 4% raise, this student will make 100% of the original salary + 4% of the original salary.

This means the new salary will be 104% of the original salary. We need to determine what part of $125 is to be taken. What part indicates percentage.

What number is

104%

of

125

Missing product statement.

|

{z

}

(percentage)

= (percent) · (base)

P

=

104%

·

125

Convert to decimals.

P

=

1.04

·

125

Multiply.

P

=

130

Thus, this student’s new monthly salary is $130.

Do Section 7.7.3.2 (Practice Set A), Exercise 7.7.4.

Example 7.41

An article of clothing is on sale at 15% o the marked price. If the marked price is $24.95, what is the sale price?

Since the item is discounted 15%, the new price will be 100% − 15% = 85% of the marked price.

We need to determine what part of 24.95 is to be taken. What part indicates percentage.

What number is

85%

of $24.95.

Missing product statement.

|

{z

}

(percentage)

= (percent) ·

(base)

P

=

85%

·

24.95

Convert to decimals.

P

=

.85

·

24.95

Multiply.

Since this number represents money,

P

=

21.2075

we’ll round to 2 decimal places

P

=

21.21

Thus, the sale price of the item is $21.21.

Available for free at Connexions <http://cnx.org/content/col10615/1.4>

451

7.7.3.2 Practice Set A

Exercise 7.7.1

(Solution on p. 471.)

What number is 42% of 85?

Exercise 7.7.2

(Solution on p. 471.)

A sales person makes a commission of 16% on each sale he makes. How much is his commission if he makes a sale of $8,500?

Exercise 7.7.3

(Solution on p. 471.)

An assembly line worker can assemble 14 parts of a product in one hour. If he can increase his assembly speed by 35%, by how many parts per hour would he increase his assembly of products?

Exercise 7.7.4

(Solution on p. 471.)

A computer scientist in the Silicon Valley makes $42,000 annually. What would this scientist’s new annual salary be if she were to receive an 8% raise?

7.7.4 Finding the Percent

7.7.4.1 Sample Set B

Example 7.42

15

is what percent of

50?

Missing factor statement.

|

{z

}

(percentage) =

(percent)

·

(base) [(product) = (factor) · (factor)]

15

=

Q

·

50

Recall that (missing factor) = (product) ÷ (known factor).

Q

=

15 ÷ 50

Divide.

Q

=

0.3

Convert to a percent.

Q

=

30%

Thus, 15 is 30% of 50.

Do Section 7.7.4.2 (Practice Set B), Exercise 7.7.5.

Example 7.43

4.32

is what percent of

72?

Missing factor statement.

|

{z

}

(percentage) =

(percent)

·

(base) [(product) = (factor) · (factor)]

4.32

=

Q

·

72

Q

=

4.32 ÷ 72

Divide.

Q

=

0.06

Convert to a percent.

Q

=

6%

Thus, 4.32 is 6% of 72.

Available for free at Connexions <http://cnx.org/content/col10615/1.4>

452

CHAPTER 7. RATIOS AND RATES

Do Section 7.7.4.2 (Practice Set B), Exercise 7.7.5.

Example 7.44

On a 160 question exam, a student got 125 correct answers. What percent is this? Round the result to two decimal places.

We need to determine the percent.

125

is what percent of

160?

Missing factor statement.

|

{z

}

(percentage) =

(percent)

·

(base) [(product) = (factor) · (factor)]

125

=

Q

·

160

Q

=

125 ÷ 160

Divide.

Q

=

0.78125

Round to two decimal places.

Q

=

.78

Thus, this student received a 78% on the exam.

Do Section 7.7.4.2 (Practice Set B), Exercise 7.7.6.

Example 7.45

A bottle contains 80 milliliters of hydrochloric acid (HCl) and 30 milliliters of water. What percent of HCl does the bottle contain? Round the result to two decimal places.

We need to determine the percent. The total amount of liquid in the bottle is 80 milliliters + 30 milliliters = 110 milliliters.

80

is what percent of

110?

Missing factor statement.

|

{z

}

(percentage) =

(percent)

·

(base) [(product) = (factor) · (factor)]

80

=

Q

·

110

Q

=

80 ÷ 110

Divide.

Q

=

0.727272. . .

Round to two decimal places.

Q

73%

The symbol “≈” is read as “approximately.”

Thus, this bottle contains approximately 73% HCl.

Do Section 7.7.4.2 (Practice Set B), Exercise 7.7.7.

Example 7.46

Five years ago a woman had an annual income of $19,200. She presently earns $42,000 annually.

By what percent has her salary increased? Round the result to two decimal places.

We need to determine the percent.

Available for free at Connexions <http://cnx.org/content/col10615/1.4>

453

42, 000

is what percent of 19, 200? Missing factor statement.

|

{z

}

(percentage) =

(percent)

·

(base)

42, 000

=

Q

·

19, 200

Q

=

42, 000 ÷ 19, 200

Divide.

Q

=

2.1875

Round to two decimal places.

Q

=

2.19

Convert to a percent.

Q

=

219%

Convert to a percent.

Thus, this woman’s annual salary has increased 219%.

7.7.4.2 Practice Set B

Exercise 7.7.5

(Solution on p. 471.)

99.13 is what percent of 431?

Exercise 7.7.6

(Solution on p. 472.)

On an 80 question exam, a student got 72 correct answers. What percent did the student get on the exam?

Exercise 7.7.7

(Solution on p. 472.)

A bottle contains 45 milliliters of sugar and 67 milliliters of water. What fraction of sugar does the bottle contain? Round the result to two decimal places (then express as a percent).

7.7.5 Finding the Base

7.7.5.1 Sample Set C

Example 7.47

15

is

30%

of what number?

Missing factor statement.

|

{z

}

(percentage) = (percent) ·

(base)

[(percentage) = (factor) · (factor)]

15

=

30%

·

B

Convert to decimals.

15

=

.30

·

B

[(missing factor) = (product) ÷ (known factor)]

B

=

15 ÷ .30

B

=

50

Thus, 15 is 30% of 50.

Try Exercise 7.7.8 in Section 7.7.5.2 (Practice Set C).

Available for free at Connexions <http://cnx.org/content/col10615/1.4>

454

CHAPTER 7. RATIOS AND RATES

Example 7.48

56.43

is

33%

of what number? Missing factor statement.

|

{z

}

(percentage) = (percent) ·

(base)

56.43

=

33%

·

B

Convert to decimals.

56.43

=

.33

·

B

Divide.

B

=

56.43 ÷ .33

B

=

171

Thus, 56.43 is 33% of 171.

Try Exercise 7.7.8 in Section 7.7.5.2 (Practice Set C).

Example 7.49

Fifteen milliliters of water represents 2% of a hydrochloric acid (HCl) solution. How many milliliters of solution are there?

We need to determine the total supply. The word supply indicates base.

15

is

2%

of what number? Missing factor statement.

|

{z

}

(percentage) = (percent) ·

(base)

15

=

2%

·

B

Convert to decimals.

15

=

.02

·

B

Divide.

B

=

15 ÷ .02

B

=

750

Thus, there are 750 milliliters of solution in the bottle.

Try Exercise 7.7.9 in Section 7.7.5.2 (Practice Set C).

Example 7.50

In a particular city, a sales tax of 61% is charged on items purchased in local stores. If the tax on 2

an item is $2.99, what is the price of the item?

We need to determine the price of the item. We can think of price as the starting place. Starting place indicates base. We need to determine the base.

2.99

is

6 1 %

of what number?

Missing factor statement.

2

|

{z

}

(percentage) = (percent) ·

(base)

2.99

=

6 1 %

·

B

Convert to decimals.

2

2.99

=

6.5%

·

B

2.99

=

.065

·

B

[(missing factor) = (product) ÷ (known factor)]

Available for free at Connexions <http://cnx.org/content/col10615/1.4>

455

B

=

2.99 ÷ .065

Divide.

B

=

46

Thus, the price of the item is $46.00.

Try Exercise 7.7.10 in Section 7.7.5.2 (Practice Set C).

Example 7.51

A clothing item is priced at $20.40. This marked price includes a 15% discount. What is the original price?

We need to determine the original price. We can think of the original price as the starting place.

Starting place indicates base. We need to determine the base. The new price, $20.40, represents 100% − 15% = 85% of the original price.

20.40

is

85%

of what number?

Missing factor statement.

|

{z

}

(percentage) = (percent) ·

(base)

20.40

=

85%

·

B

Convert to decimals.

20.40

=

.85

·

B

[(missing factor) = (product) ÷ (known factor)]

B

=

20.40 ÷ .85

Divide.

B

=

24

Thus, the original price of the item is $24.00.

Try Exercise 7.7.11 in Section 7.7.5.2 (Practice Set C).

7.7.5.2 Practice Set C

Exercise 7.7.8

(Solution on p. 472.)

1.98 is 2% of what number?

Exercise 7.7.9

(Solution on p. 472.)

3.3 milliliters of HCl represents 25% of an HCl solution. How many milliliters of solution are there?

Exercise 7.7.10

(Solution on p. 472.)

A salesman, who makes a commission of 181% on each sale, makes a commission of $152.39 on a 4

particular sale. Rounded to the nearest dollar, what is the amount of the sale?

Exercise 7.7.11

(Solution on p. 472.)

At “super-long play,” 21hours of play of a video cassette recorder represents 31.25% of the total 2

playing time. What is the total playing time?

7.7.6 Exercises

For the following 25 problems, nd each indicated quantity.

Exercise 7.7.12

(Solution on p. 472.)

What is 21% of 104?

Available for free at Connexions <http://cnx.org/content/col10615/1.4>

456

CHAPTER 7. RATIOS AND RATES

Exercise 7.7.13

What is 8% of 36?

Exercise 7.7.14

(Solution on p. 472.)

What is 98% of 545?

Exercise 7.7.15

What is 143% of 33?

Exercise 7.7.16

(Solution on p. 472.)

What is 101% of 20?

2

Exercise 7.7.17

3.25 is what percent of 88?

Exercise 7.7.18

(Solution on p. 472.)

22.44 is what percent of 44?

Exercise 7.7.19

0.0036 is what percent of 0.03?

Exercise 7.7.20

(Solution on p. 472.)

31.2 is what percent of 26?

Exercise 7.7.21

266.4 is what percent of 74?

Exercise 7.7.22

(Solution on p. 472.)

0.0101 is what percent of 0.0505?

Exercise 7.7.23

2.4 is 24% of what number?

Exercise 7.7.24

(Solution on p. 472.)

24.19 is 41% of what number?

Exercise 7.7.25

61.12 is 16% of what number?

Exercise 7.7.26

(Solution on p. 472.)

82.81 is 91% of what number?

Exercise 7.7.27

115.5 is 20% of what number?

Exercise 7.7.28

(Solution on p. 472.)

43.92 is 480% of what number?

Exercise 7.7.29

What is 85% of 62?

Exercise 7.7.30

(Solution on p. 472.)

29.14 is what percent of 5.13?

Exercise 7.7.31

0.6156 is what percent of 5.13?

Exercise 7.7.32

(Solution on p. 472.)

What is 0.41% of 291.1?

Exercise 7.7.33

26.136 is 121% of what number?

Exercise 7.7.34

(Solution on p. 472.)

1,937.5 is what percent of 775?

Exercise 7.7.35

1 is what percent of 2,000?

Available for free at Connexions <http://cnx.org/content/col10615/1.4>

457

Exercise 7.7.36

(Solution on p. 472.)

0 is what percent of 59?

Exercise 7.7.37

An item of clothing is on sale for 10% o the marked price. If the marked price is $14.95, what is the sale price? (Round to two decimal places.)

Exercise 7.7.38

(Solution on p. 472.)

A grocery clerk, who makes $365 per month, receives a 7% raise. How much is her new monthly salary?

Exercise 7.7.39

An item of clothing which originally sells for $55.00 is marked down to $46.75. What percent has it been marked down?

Exercise 7.7.40

(Solution on p. 472.)

On a 25 question exam, a student gets 21 correct. What percent is this?

Exercise 7.7.41

On a 45 question exam, a student gets 40%. How many questions did this student get correct?

Exercise 7.7.42

(Solution on p. 472.)

A vitamin tablet, which weighs 250 milligrams, contains 35 milligrams of vitamin C. What percent of the weight of this tablet is vitamin C?

Exercise 7.7.43

Five years ago a secretary made $11,200 annually. The secretary now makes $17,920 annually. By what percent has this secretary’s salary been increased?

Exercise 7.7.44

(Solution on p. 472.)

A baseball team wins 483% of all their games. If they won 78 games, how many games did they 4

play?

Exercise 7.7.45

A typist was able to increase his speed by 120% to 42 words per minute. What was his original typing speed?

Exercise 7.7.46

(Solution on p. 472.)

A salesperson makes a commission of 12% on the total amount of each sale. If, in one month, she makes a total of $8,520 in sales, how much has she made in commission?

Exercise 7.7.47

A salesperson receives a salary of $850 per month plus a commission of 81% of her sales. If, in a 2

particular month, she sells $22,800 worth of merchandise, what will be her monthly earnings?

Exercise 7.7.48

(Solution on p. 472.)

A man borrows $1150.00 from a loan company. If he makes 12 equal monthly payments of $130.60, what percent of the loan is he paying in interest?

Exercise 7.7.49

The distance from the sun to the earth is approximately 93,000,000 miles. The distance from the sun to Pluto is approximately 860.2% of the distance from the sun to the Earth. Approximately, how many miles is Pluto from the sun?

Exercise 7.7.50

(Solution on p. 473.)

The number of people on food stamps in Maine in 1975 was 151,000. By 1980, the number had decreased to 59,200. By what percent did the number of people on food stamps decrease? (Round the result to the nearest percent.)

Exercise 7.7.51

In Nebraska, in 1960, there were 734,000 motor-vehicle registrations. By 1979, the total had increased by about 165.6%. About how many motor-vehicle registrations were there in Nebraska in 1979?

Available for free at Connexions <http://cnx.org/content/col10615/1.4>

458

CHAPTER 7. RATIOS AND RATES

Exercise 7.7.52

(Solution on p. 473.)

From 1973 to 1979, in the United States, there was an increase of 166.6% of Ph.D. social scientists to 52,000. How many were there in 1973?

Exercise 7.7.53

In 1950, in the United States, there were 1,894 daily newspapers. That number decreased to 1,747

by 1981. What percent did the number of daily newspapers decrease?

Exercise 7.7.54

(Solution on p. 473.)

A particular alloy is 27% copper. How many pounds of copper are there in 55 pounds of the alloy?

Exercise 7.7.55

A bottle containing a solution of hydrochloric acid (HCl) is marked 15% (meaning that 15% of the HCl solution is acid). If a bottle contains 65 milliliters of solution, how many milliliters of water does it contain?

Exercise 7.7.56

(Solution on p. 473.)

A bottle containing a solution of HCl is marked 45%. A test shows that 36 of the 80 milliliters contained in the bottle are hydrochloric acid. Is the bottle marked correctly? If not, how should it be remarked?

7.7.6.1 Exercises For Review

Exercise 7.7.57

(Section 2.6) Use the numbers 4 and 7 to illustrate the commutative property of multiplication.

Exercise 7.7.58

(Solution on p. 473.)

(Section 4.3) Convert 14 to a mixed number.

5

Exercise 7.7.59

(Section 5.5) Arrange the numbers 712, 5 and 4 in increasing order.

9

7

Exercise 7.7.60

(Solution on p. 473.)

(Section 6.3) Convert 4.006 to a mixed number.

Exercise 7.7.61

(Section 7.6) Convert 7 % to a fraction.

8

Available for free at Connexions <http://cnx.org/content/col10615/1.4>

459

7.8 Summary of Key Concepts8

7.8.1 Summary of Key Concepts

Denominate Numbers (Section 7.2)

Numbers that appear along with units are denominate numbers. The amounts 6 dollars and 4 pints are examples of denominate numbers.

Like and Unlike Denominate Numbers (Section 7.2)

Like denominate numbers are denominate numbers with like units. If the units are not the same, the numbers are unlike denominate numbers.

Pure Numbers (Section 7.2)

Numbers appearing without a unit are pure numbers.

Comparing Numbers by Subtraction and Division (Section 7.2)

Comparison of two numbers by subtraction indicates how much more one number is than another. Comparison by division indicates how many times larger or smaller one number is than another.

Comparing Pure or Like Denominate Numbers by Subtraction (Section 7.2)

Numbers can be compared by subtraction if and only if they are pure numbers or like denominate numbers.

Ratio Rate (Section 7.2)

A comparison, by division, of two like denominate numbers is a ratio. A comparison, by division, of two unlike denominate numbers is a rate.

Proportion (Section 7.3)

A proportion is a statement that two ratios or rates are equal.

3 people

2 jobs

= 6 people

4 jobs

is a proportion.

Solving a Proportion (Section 7.3)

To solve a proportion that contains three known numbers and a letter that represents an unknown quantity, perform the cross multiplication, then divide the product of the two numbers by the number that multiplies the letter.

Proportions Involving Rates (Section 7.3)

When writing a proportion involving rates it is very important to write it so that the same type of units appears on the same side of either the equal sign or the fraction bar.

unit type 1

unit type 2 = unit type 1

unit type 2 or unit type 1

unit type 1 = unit type 2

unit type 2

Five-Step Method for Solving Proportions (Section 7.4)

1. By careful reading, determine what the unknown quantity is and represent it with some letter. There will be only one unknown in a problem.

2. Identify the three specied numbers.

3. Determine which comparisons are to be made and set up the proportion.

4. Solve the proportion.

5. Interpret and write a conclusion.

When solving applied problems, ALWAYS begin by determining the unknown quantity and representing it with a letter.

8This content is available online at <http://cnx.org/content/m35008/1.2/>.

Available for free at Connexions <http://cnx.org/content/col10615/1.4>

460

CHAPTER 7. RATIOS AND RATES

Percents (Section 7.5)

A ratio in which one number is compared to 100 is a percent. Percent means “for each hundred.”

Conversion of Fractions, Decimals, and Percents (Section 7.5)

It is possible to convert decimals to percents, fractions to percents, percents to decimals, and percents to fractions.

Applications of Percents:

The three basic types of percent problems involve a base, a percentage, and a percent.

Base (Section 7.7)

The base is the number used for comparison.

Percentage (Section 7.7)

The percentage is the number being compared to the base.

Percent (Section 7.7)

By its denition, percent means part of.

Solving Problems (Section 7.7)

Percentage = (percent) × (base)

Percent = percentage

base

Base = percentage

percent

7.9 Exercise Supplement9

7.9.1 Exercise Supplement

7.9.1.1 Ratios and Rates (Section 7.2)

Exercise 7.9.1

(Solution on p. 473.)

Compare 250 watts to 100 watts by subtraction.

Exercise 7.9.2

Compare 126 and 48 by subtraction.

Exercise 7.9.3

(Solution on p. 473.)

Compare 98 radishes to 41 radishes by division.

Exercise 7.9.4

Compare 144 to 9 by division.

Exercise 7.9.5

(Solution on p. 473.)

Compare 100 tents to 5 tents by division.

Exercise 7.9.6

Compare 28 feet to 7 feet by division.

Exercise 7.9.7

(Solution on p. 473.)

Comparison, by division, of two pure numbers or two like denominate numbers is called a

.

Exercise 7.9.8

A comparison, by division, of two unlike denominate numbers is called a

.

For problems 9-12, express each ratio or rate as a fraction.

9This content is available online at <http://cnx.org/content/m35009/1.2/>.

Available for free at Connexions <http://cnx.org/content/col10615/1.4>

461

Exercise 7.9.9

(Solution on p. 473.)

15 to 5

Exercise 7.9.10

72 to 12

Exercise 7.9.11

(Solution on p. 473.)

8 millimeters to 5 milliliters

Exercise 7.9.12

106 tablets to 52 tablets

For problems 13-16, write each ratio in the form “a to b”.

Exercise 7.9.13

(Solution on p. 473.)

9

16

Exercise 7.9.14

5

11

Exercise 7.9.15

(Solution on p. 473.)

1 diskette

8 diskettes

Exercise 7.9.16

5 papers

3 pens

For problems 17-21, write each ratio or rate using words.

Exercise 7.9.17

(Solution on p. 473.)

9 = 18

16

32

Exercise 7.9.18

1 = 12

4

48

Exercise 7.9.19

(Solution on p. 473.)

8 items

4 dollars = 2 items

1 dollar

Exercise 7.9.20

150 milligrams of niacin is to 2 tablets as 300 milligrams of niacin is to 4 tablets.

Exercise 7.9.21

(Solution on p. 473.)

20 people is to 4 seats as 5 people is to 1 seat.

7.9.1.2 Proportions (Section 7.3)

For problems 22-27, determine the missing number in each proportion.

Exercise 7.9.22

x = 24

3

9

Exercise 7.9.23

(Solution on p. 473.)

15 = 60

7

x

Exercise 7.9.24

1 = x

1

44

Exercise 7.9.25

(Solution on p. 473.)

3 = 15

x

50

Exercise 7.9.26

15 bats

16 balls =

x bats

128 balls

Exercise 7.9.27

(Solution on p. 473.)

36 rooms

29 fans

= 504 rooms

x fans

Available for free at Connexions <http://cnx.org/content/col10615/1.4>

462

CHAPTER 7. RATIOS AND RATES

7.9.1.3 Applications of Proportions (Section 7.4)

Exercise 7.9.28

On a map, 3 inches represents 20 miles. How many miles does 27 inches represent?

Exercise 7.9.29

(Solution on p. 473.)

A salt solution is composed of 8 parts of salt to 5 parts of water. How many parts of salt are there in a solution that contains 50 parts of water?

Exercise 7.9.30

A model is built to 4 scale. If a particular part of the model measures 8 inches in length, how long 15

is the actual structure?

Exercise 7.9.31

(Solution on p. 473.)

The ratio of ammonia to air in a container is 3 How many milliliters of air should be in a container 40

that contains 8 milliliters of ammonia?

Exercise 7.9.32

A 4-foot girl casts a 9-foot shadow at a particular time of the day. How tall is a pole that casts a 144-foot shadow at the same time of the day?

Exercise 7.9.33

(Solution on p. 473.)

The odds that a particular event will occur are 11 to 2. If this event occurs 55 times, how many times would you predict it does not occur?

Exercise 7.9.34

Every 1 3 teaspoon of a multiple vitamin, in granular form, contains 0.85 the minimum daily 4

requirement of vitamin A. How many teaspoons of this vitamin are required to supply 2.25 the minimum daily requirement?

7.9.1.4 Percent and Fractions of One Percent (Section 7.5,Section 7.6)

For problems 35-39, convert each decimal to a percent.

Exercise 7.9.35

(Solution on p. 473.)

0.16

Exercise 7.9.36

0.818

Exercise 7.9.37

(Solution on p. 473.)

5.3536

Exercise 7.9.38

0.50

Exercise 7.9.39

(Solution on p. 474.)

3

For problems 40-48, convert each percent to a decimal.

Exercise 7.9.40

62%

Exercise 7.9.41

(Solution on p. 474.)

1.58%

Exercise 7.9.42

9.15%

Exercise 7.9.43

(Solution on p. 474.)

0.06%

Available for free at Connexions <http://cnx.org/content/col10615/1.4>

463

Exercise 7.9.44

0.003%

Exercise 7.9.45

(Solution on p. 474.)

5 3 % to a three-place decimal

11

Exercise 7.9.46

9

% to a three-place decimal

13

Exercise 7.9.47

(Solution on p. 474.)

82 25 % to a four-place decimal

29

Exercise 7.9.48

18 1 % to a four-place decimal

7

For problems 49-55, convert each fraction or mixed number to a percent.

Exercise 7.9.49

(Solution on p. 474.)

3

5

Exercise 7.9.50

2

10

Exercise 7.9.51

(Solution on p. 474.)

5

16

Exercise 7.9.52

35

8

Exercise 7.9.53

(Solution on p. 474.)

105

16

Exercise 7.9.54

45 111

Exercise 7.9.55

(Solution on p. 474.)

6 278

9

For problems 56-64, convert each percent to a fraction or mixed number.

Exercise 7.9.56

95%

Exercise 7.9.57

(Solution on p. 474.)

12%

Exercise 7.9.58

83%

Exercise 7.9.59

(Solution on p. 474.)

38.125%

Exercise 7.9.60

_

61. 2 %

Exercise 7.9.61

(Solution on p. 474.)

5 %

8

Exercise 7.9.62

6 9 %

20

Exercise 7.9.63

(Solution on p. 474.)

15 3 %

22

Exercise 7.9.64

106 19 %

45

Available for free at Connexions <http://cnx.org/content/col10615/1.4>

464

CHAPTER 7. RATIOS AND RATES

7.9.1.5 Applications of Percents (Section 7.7)

For problems 65-72, nd each solution.

Exercise 7.9.65

(Solution on p. 474.)

What is 16% of 40?

Exercise 7.9.66

29.4 is what percent of 105?

Exercise 7.9.67

(Solution on p. 474.)

3 21 is 547.2% of what number?

50

Exercise 7.9.68

0.09378 is what percent of 52.1?

Exercise 7.9.69

(Solution on p. 474.)

What is 680% of 1.41?

Exercise 7.9.70

A kitchen knife is on sale for 15% o the marked price. If the marked price is $ 39.50, what is the sale price?

Exercise 7.9.71

(Solution on p. 474.)

On an 80 question geology exam, a student gets 68 correct. What percent is correct?

Exercise 7.9.72

A salesperson makes a commission of 18% of her monthly sales total. She also receives a monthly salary of $1,600.00. If, in a particular month, she sells $4,000.00 worth of merchandise, how much will she make that month?

7.10 Prociency Exam10

7.10.1 Prociency Exam

Exercise 7.10.1

(Solution on p. 474.)

(Section 7.2) Compare 4 cassette tapes to 7 dollars.

Exercise 7.10.2

(Solution on p. 474.)

(Section 7.2) What do we call a comparison, by division, of two unlike denominate numbers?

For problems 3 and 4, express each ratio or rate as a fraction.

Exercise 7.10.3

(Solution on p. 474.)

(Section 7.2) 11 to 9

Exercise 7.10.4

(Solution on p. 474.)

(Section 7.2) 5 televisions to 2 radios

For problems 5 and 6, write each ratio or rate in the form “a to b.”

Exercise 7.10.5

(Solution on p. 474.)

(Section 7.2) 8 maps

3 people

Exercise 7.10.6

(Solution on p. 474.)

(Section 7.2) 2 psychologists

75 people

For problems 7-9, solve each proportion.

Exercise 7.10.7

(Solution on p. 474.)

(Section 7.3) 8 = 48

x

90

10This content is available online at <http://cnx.org/content/m35010/1.2/>.

Available for free at Connexions <http://cnx.org/content/col10615/1.4>

465

Exercise 7.10.8

(Solution on p. 474.)

(Section 7.3) x = 4

7

28

Exercise 7.10.9

(Solution on p. 475.)

(Section 7.3) 3 computers

8 students = 24 computers

x students

Exercise 7.10.10

(Solution on p. 475.)

(Section 7.4) On a map, 4 inches represents 50 miles. How many miles does 3 inches represent?

Exercise 7.10.11

(Solution on p. 475.)

(Section 7.4) An acid solution is composed of 6 milliliters of acid to 10 milliliters of water. How many milliliters of acid are there in an acid solution that is composed of 3 milliliters of water?

Exercise 7.10.12

(Solution on p. 475.)

(Section 7.4) The odds that a particular event will occur are 9 to 7. If the event occurs 27 times, how many times would you predict it will it not occur?

For problems 13 and 14, convert each decimal to a percent.

Exercise 7.10.13

(Solution on p. 475.)

(Section 7.5) 0.82

Exercise 7.10.14

(Solution on p. 475.)

_

(Section 7.5) 3. 7

For problems 15 and 16, convert each percent to a decimal.

Exercise 7.10.15

(Solution on p. 475.)

(Section 7.5) 2.813%

Exercise 7.10.16

(Solution on p. 475.)

(Section 7.5) 0.006%

For problems 17-19, convert each fraction to a percent.

Exercise 7.10.17

(Solution on p. 475.)

(Section 7.5) 425

Exercise 7.10.18

(Solution on p. 475.)

(Section 7.5) 18

Exercise 7.10.19

(Solution on p. 475.)

(Section 7.5) 800

80

For problems 20 and 21, convert each percent to a fraction.

Exercise 7.10.20

(Solution on p. 475.)

(Section 7.5) 15%

Exercise 7.10.21

(Solution on p. 475.)

(Section 7.5) 427 %

For problems 22-25, nd each indicated quantity.

Exercise 7.10.22

(Solution on p. 475.)

(Section 7.7) What is 18% of 26?

Exercise 7.10.23

(Solution on p. 475.)

(Section 7.7) 0.618 is what percent of 0.3?

Exercise 7.10.24

(Solution on p. 475.)

(Section 7.7) 0.1 is 1.1% of what number?

Exercise 7.10.25

(Solution on p. 475.)

(Section 7.7) A salesperson makes a monthly salary of $1,000.00. He also gets a commission of 12% of his total monthly sales. If, in a particular month, he sells $5,500.00 worth of merchandise, what is his income that month?

Available for free at Connexions <http://cnx.org/content/col10615/1.4>

466

CHAPTER 7. RATIOS AND RATES

Solutions to Exercises in Chapter 7

Solution to Exercise 7.2.1 (p. 421)

(a) 8 diskettes; 10 diskettes is 8 diskettes more than 2 diskettes.

(b) 5; 10 diskettes is 5 times as many diskettes as 2 diskettes.

Solution to Exercise 7.2.2 (p. 421)

(a) Comparison by subtraction makes no sense.

(b) 16 bananas

2 bags

= 8 bananas

bag

, 8 bananas per bag.

Solution to Exercise 7.2.3 (p. 422)

3

2

Solution to Exercise 7.2.4 (p. 422)

1

9

Solution to Exercise 7.2.5 (p. 422)

5 books

4 people

Solution to Exercise 7.2.6 (p. 422)

60 miles

1 hour

Solution to Exercise 7.2.7 (p. 422)

8

3

Solution to Exercise 7.2.8 (p. 422)

9 to 5

Solution to Exercise 7.2.9 (p. 422)

1 to 3

Solution to Exercise 7.2.10 (p. 422)

25 miles to 2 gallons

Solution to Exercise 7.2.11 (p. 423)

1 mechanic to 2 wrenches

Solution to Exercise 7.2.12 (p. 423)

5 to 6

Solution to Exercise 7.2.13 (p. 423)

They are pure numbers or like denominate numbers.

Solution to Exercise 7.2.15 (p. 423)

rate

Solution to Exercise 7.2.17 (p. 423)

ratio

Solution to Exercise 7.2.19 (p. 423)

rate

Solution to Exercise 7.2.21 (p. 423)

ratio

Solution to Exercise 7.2.23 (p. 423)

two to ve

Solution to Exercise 7.2.25 (p. 423)

29 mile per 2 gallons or 141 miles per 1 gallon

2

Solution to Exercise 7.2.27 (p. 423)

5 to 2

Solution to Exercise 7.2.29 (p. 423)

12

5

Solution to Exercise 7.2.31 (p. 424)

42 plants

5 homes

Available for free at Connexions <http://cnx.org/content/col10615/1.4>

467

Solution to Exercise 7.2.33 (p. 424)

16 pints

1 quart

Solution to Exercise 7.2.35 (p. 424)

2.54 cm

1 inch

Solution to Exercise 7.2.37 (p. 424)

5

2

Solution to Exercise 7.2.39 (p. 424)

1 hit

3 at bats

Solution to Exercise 7.2.41 (p. 424)

1,042 characters

1 page

Solution to Exercise 7.2.43 (p. 424)

5 13

Solution to Exercise 7.2.45 (p. 424)

299

1260

Solution to Exercise 7.2.47 (p. 424)

3.3875

Solution to Exercise 7.3.1 (p. 426)

3 is to 8 as 6 is to 16

Solution to Exercise 7.3.2 (p. 426)

2 people are to 1 window as 10 people are to 5 windows

Solution to Exercise 7.3.3 (p. 426)

15 = 75

4

20

Solution to Exercise 7.3.4 (p. 426)

2 plates

1 tray = 20 plates

10 trays

Solution to Exercise 7.3.5 (p. 427)

x = 3

Solution to Exercise 7.3.6 (p. 428)

x = 5

Solution to Exercise 7.3.7 (p. 428)

x = 45

Solution to Exercise 7.3.8 (p. 428)

x = 48

Solution to Exercise 7.3.9 (p. 429)

rates, proportion

Solution to Exercise 7.3.11 (p. 429)

1

11 = 3

33

Solution to Exercise 7.3.13 (p. 429)

6

90 = 3

45

Solution to Exercise 7.3.15 (p. 429)

18 gr cobalt

10 gr silver = 36 gr cobalt

20 gr silver

Solution to Exercise 7.3.17 (p. 429)

3 people absent

31 people present = 15 people absent

155 people present

Solution to Exercise 7.3.19 (p. 429)

3 is to 4 as 15 is to 20

Solution to Exercise 7.3.21 (p. 429)

3 joggers are to 100 feet as 6 joggers are to 200 feet

Solution to Exercise 7.3.23 (p. 430)

40 miles are to 80 miles as 2 gallons are to 4 gallons

Solution to Exercise 7.3.25 (p. 430)

1 person is to 1 job as 8 people are to 8 jobs

Available for free at Connexions <http://cnx.org/content/col10615/1.4>

468

CHAPTER 7. RATIOS AND RATES

Solution to Exercise 7.3.27 (p. 430)

2,000 pounds are to 1 ton as 60,000 pounds are to 30 tons

Solution to Exercise 7.3.29 (p. 430)

x = 2

Solution to Exercise 7.3.31 (p. 430)

x = 8

Solution to Exercise 7.3.33 (p. 430)

x = 8

Solution to Exercise 7.3.35 (p. 430)

x = 27

Solution to Exercise 7.3.37 (p. 430)

x = 40

Solution to Exercise 7.3.39 (p. 430)

x = 30

Solution to Exercise 7.3.41 (p. 430)

x = 35

Solution to Exercise 7.3.43 (p. 430)

x = 30

Solution to Exercise 7.3.45 (p. 431)

true

Solution to Exercise 7.3.47 (p. 431)

false

Solution to Exercise 7.3.49 (p. 431)

true

Solution to Exercise 7.3.51 (p. 431)

true

Solution to Exercise 7.3.53 (p. 431)

5+7=12

7+5=12

Solution to Exercise 7.3.55 (p. 431)

17

77

Solution to Exercise 7.3.57 (p. 431)

8 sentences

1 paragraph

Solution to Exercise 7.4.1 (p. 434)

500 miles

Solution to Exercise 7.4.2 (p. 435)

24 parts of alcohol

Solution to Exercise 7.4.3 (p. 435)

3 9

11 feet

Solution to Exercise 7.4.4 (p. 435)

864 outside plants

Solution to Exercise 7.4.5 (p. 435)

The event occurs 99 times.

Solution to Exercise 7.4.6 (p. 435)

6 failing grades

Solution to Exercise 7.4.7 (p. 436)

24

Solution to Exercise 7.4.9 (p. 436)

45 inches

Available for free at Connexions <http://cnx.org/content/col10615/1.4>

469

Solution to Exercise 7.4.11 (p. 436)

33 parts

Solution to Exercise 7.4.13 (p. 436)

2328.75

Solution to Exercise 7.4.15 (p. 436)

21

22 feet

Solution to Exercise 7.4.17 (p. 436)

12,250

Solution to Exercise 7.4.19 (p. 436)

12

Solution to Exercise 7.4.21 (p. 436)

1 56

Solution to Exercise 7.4.23 (p. 437)

1 78

Solution to Exercise 7.4.25 (p. 437)

3114

Solution to Exercise 7.4.27 (p. 437)

0Solution to Exercise 7.4.29 (p. 437)

2

3

Solution to Exercise 7.4.31 (p. 437)

5Solution to Exercise 7.5.1 (p. 438)

16%

Solution to Exercise 7.5.2 (p. 438)

195%

Solution to Exercise 7.5.3 (p. 438)

83

100

Solution to Exercise 7.5.4 (p. 438)

362

100 or 181

50

Solution to Exercise 7.5.5 (p. 441)

0.21

Solution to Exercise 7.5.6 (p. 441)

4.61

Solution to Exercise 7.5.7 (p. 441)

55%

Solution to Exercise 7.5.8 (p. 441)

564%

Solution to Exercise 7.5.9 (p. 441)

15%

Solution to Exercise 7.5.10 (p. 441)

137.5%

Solution to Exercise 7.5.11 (p. 441)

27.27%

Solution to Exercise 7.5.12 (p. 441)

25%

Solution to Exercise 7.5.14 (p. 441)

48%

Solution to Exercise 7.5.16 (p. 441)

77.1%

Available for free at Connexions <http://cnx.org/content/col10615/1.4>

470

CHAPTER 7. RATIOS AND RATES

Solution to Exercise 7.5.18 (p. 441)

258%

Solution to Exercise 7.5.20 (p. 441)

1,618.14%

Solution to Exercise 7.5.22 (p. 441)

200%

Solution to Exercise 7.5.24 (p. 442)

0.15

Solution to Exercise 7.5.26 (p. 442)

0.162

Solution to Exercise 7.5.28 (p. 442)

0.0505

Solution to Exercise 7.5.30 (p. 442)

0.0078

Solution to Exercise 7.5.32 (p. 442)

0.0009

Solution to Exercise 7.5.34 (p. 442)

20%

Solution to Exercise 7.5.36 (p. 442)

62.5%

Solution to Exercise 7.5.38 (p. 442)

28%

Solution to Exercise 7.5.40 (p. 442)

49.09%

Solution to Exercise 7.5.42 (p. 442)

164%

Solution to Exercise 7.5.44 (p. 442)

945%

Solution to Exercise 7.5.46 (p. 443)

54.54%

Solution to Exercise 7.5.48 (p. 443)

4

5

Solution to Exercise 7.5.50 (p. 443)

1

4

Solution to Exercise 7.5.52 (p. 443)

13

20

Solution to Exercise 7.5.54 (p. 443)

1

8

Solution to Exercise 7.5.56 (p. 443)

41 or 51

8

8

Solution to Exercise 7.5.58 (p. 443)

1

10

Solution to Exercise 7.5.60 (p. 443)

2

9

Solution to Exercise 7.5.62 (p. 443)

4

45

Solution to Exercise 7.5.64 (p. 443)

129

60 or 2 9

60 = 2 3

20

Solution to Exercise 7.5.66 (p. 444)

36 inches

Available for free at Connexions <http://cnx.org/content/col10615/1.4>

471

Solution to Exercise 7.6.1 (p. 445)

1

400

Solution to Exercise 7.6.2 (p. 445)

3

800

Solution to Exercise 7.6.3 (p. 445)

1

30

Solution to Exercise 7.6.4 (p. 446)

0.008

Solution to Exercise 7.6.5 (p. 446)

0.5145

Solution to Exercise 7.6.6 (p. 446)

3

400

Solution to Exercise 7.6.8 (p. 446)

1

900

Solution to Exercise 7.6.10 (p. 446)

5

400 or 1

80

Solution to Exercise 7.6.12 (p. 446)

13

700

Solution to Exercise 7.6.14 (p. 446)

101

400

Solution to Exercise 7.6.16 (p. 446)

363

500

Solution to Exercise 7.6.18 (p. 446)

41

30

Solution to Exercise 7.6.20 (p. 447)

51

500 = 0.102

Solution to Exercise 7.6.22 (p. 447)

31

800 = 0.03875

Solution to Exercise 7.6.24 (p. 447)

0.004

Solution to Exercise 7.6.26 (p. 447)

0.0627

Solution to Exercise 7.6.28 (p. 447)

0.242

Solution to Exercise 7.6.30 (p. 447)

0.1194

Solution to Exercise 7.6.32 (p. 447)

85

Solution to Exercise 7.6.34 (p. 447)

197

210

Solution to Exercise 7.6.36 (p. 447)

806.2%

Solution to Exercise 7.7.1 (p. 451)

35.7

Solution to Exercise 7.7.2 (p. 451)

$1,360

Solution to Exercise 7.7.3 (p. 451)

4.9

Solution to Exercise 7.7.4 (p. 451)

$45,360

Available for free at Connexions <http://cnx.org/content/col10615/1.4>

472

CHAPTER 7. RATIOS AND RATES

Solution to Exercise 7.7.5 (p. 453)

23%

Solution to Exercise 7.7.6 (p. 453)

90%

Solution to Exercise 7.7.7 (p. 453)

40%

Solution to Exercise 7.7.8 (p. 455)

99

Solution to Exercise 7.7.9 (p. 455)

13.2ml

Solution to Exercise 7.7.10 (p. 455)

$835

Solution to Exercise 7.7.11 (p. 455)

8 hours

Solution to Exercise 7.7.12 (p. 455)

21.84

Solution to Exercise 7.7.14 (p. 456)

534.1

Solution to Exercise 7.7.16 (p. 456)

2.1

Solution to Exercise 7.7.18 (p. 456)

51

Solution to Exercise 7.7.20 (p. 456)

120

Solution to Exercise 7.7.22 (p. 456)

20

Solution to Exercise 7.7.24 (p. 456)

59

Solution to Exercise 7.7.26 (p. 456)

91

Solution to Exercise 7.7.28 (p. 456)

9.15

Solution to Exercise 7.7.30 (p. 456)

568

Solution to Exercise 7.7.32 (p. 456)

1.19351

Solution to Exercise 7.7.34 (p. 456)

250

Solution to Exercise 7.7.36 (p. 457)

0Solution to Exercise 7.7.38 (p. 457)

390.55

Solution to Exercise 7.7.40 (p. 457)

84

Solution to Exercise 7.7.42 (p. 457)

14

Solution to Exercise 7.7.44 (p. 457)

160

Solution to Exercise 7.7.46 (p. 457)

$1,022.40

Available for free at Connexions <http://cnx.org/content/col10615/1.4>

473

Solution to Exercise 7.7.48 (p. 457)

36.28%

Solution to Exercise 7.7.50 (p. 457)

61

Solution to Exercise 7.7.52 (p. 458)

19,500

Solution to Exercise 7.7.54 (p. 458)

14.85

Solution to Exercise 7.7.56 (p. 458)

Marked correctly

Solution to Exercise 7.7.58 (p. 458)

2 45

Solution to Exercise 7.7.60 (p. 458)

4 3

500

Solution to Exercise 7.9.1 (p. 460)

250 watts are 150 watts more than 100 watts

Solution to Exercise 7.9.3 (p. 460)

98 radishes are 2.39 times as many radishes as 41 radishes

Solution to Exercise 7.9.5 (p. 460)

100 tents are 20 times as many tents as 5 tents

Solution to Exercise 7.9.7 (p. 460)

ratio

Solution to Exercise 7.9.9 (p. 460)

3

1

Solution to Exercise 7.9.11 (p. 461)

8ml

5ml

Solution to Exercise 7.9.13 (p. 461)

9 to 16

Solution to Exercise 7.9.15 (p. 461)

1 diskette to 8 diskettes

Solution to Exercise 7.9.17 (p. 461)

9 is to 16 as 18 is to 32

Solution to Exercise 7.9.19 (p. 461)

8 items are to 4 dollars as 2 items are to 1 dollar

Solution to Exercise 7.9.21 (p. 461)

20 = 5

4

1

20 people are to 4 seats as 5 people are to 1 seat

Solution to Exercise 7.9.23 (p. 461)

28

Solution to Exercise 7.9.25 (p. 461)

10

Solution to Exercise 7.9.27 (p. 461)

406

Solution to Exercise 7.9.29 (p. 462)

80

Solution to Exercise 7.9.31 (p. 462)

320 or 1062

3

3

Solution to Exercise 7.9.33 (p. 462)

10

Solution to Exercise 7.9.35 (p. 462)

16%

Available for free at Connexions <http://cnx.org/content/col10615/1.4>

474

CHAPTER 7. RATIOS AND RATES

Solution to Exercise 7.9.37 (p. 462)

535.36%

Solution to Exercise 7.9.39 (p. 462)

300%

Solution to Exercise 7.9.41 (p. 462)

0.0158

Solution to Exercise 7.9.43 (p. 462)

0.0006

Solution to Exercise 7.9.45 (p. 463)

0.053

Solution to Exercise 7.9.47 (p. 463)

0.8286

Solution to Exercise 7.9.49 (p. 463)

60%

Solution to Exercise 7.9.51 (p. 463)

31.25%

Solution to Exercise 7.9.53 (p. 463)

656.25%

Solution to Exercise 7.9.55 (p. 463)

_

3688. 8 %

Solution to Exercise 7.9.57 (p. 463)

3

25

Solution to Exercise 7.9.59 (p. 463)

61

160

Solution to Exercise 7.9.61 (p. 463)

1

160

Solution to Exercise 7.9.63 (p. 463)

2977

19800

Solution to Exercise 7.9.65 (p. 464)

6.4

Solution to Exercise 7.9.67 (p. 464)

0.625 or 58

Solution to Exercise 7.9.69 (p. 464)

9.588

Solution to Exercise 7.9.71 (p. 464)

85

Solution to Exercise 7.10.1 (p. 464)

4 cassette tapes

7 dollars

Solution to Exercise 7.10.2 (p. 464)

Rate

Solution to Exercise 7.10.3 (p. 464)

11

9

Solution to Exercise 7.10.4 (p. 464)

5 televisions

2 radios

Solution to Exercise 7.10.5 (p. 464)

8 maps to 3 people

Solution to Exercise 7.10.6 (p. 464)

two psychologists to seventy-ve people

Solution to Exercise 7.10.7 (p. 464)

15

Available for free at Connexions <http://cnx.org/content/col10615/1.4>

475

Solution to Exercise 7.10.8 (p. 465)

1Solution to Exercise 7.10.9 (p. 465)

64

Solution to Exercise 7.10.10 (p. 465)

3712

Solution to Exercise 7.10.11 (p. 465)

1.8

Solution to Exercise 7.10.12 (p. 465)

21

Solution to Exercise 7.10.13 (p. 465)

82%

Solution to Exercise 7.10.14 (p. 465)

3777%

9

Solution to Exercise 7.10.15 (p. 465)

0.02813

Solution to Exercise 7.10.16 (p. 465)

0.00006

Solution to Exercise 7.10.17 (p. 465)

840%

Solution to Exercise 7.10.18 (p. 465)

12.5%

Solution to Exercise 7.10.19 (p. 465)

1,000%

Solution to Exercise 7.10.20 (p. 465)

3

20

Solution to Exercise 7.10.21 (p. 465)

4

2,700 or 1

675

Solution to Exercise 7.10.22 (p. 465)

4.68

Solution to Exercise 7.10.23 (p. 465)

206

Solution to Exercise 7.10.24 (p. 465)

9.09

Solution to Exercise 7.10.25 (p. 465)

$1,660

Available for free at Connexions <http://cnx.org/content/col10615/1.4>

476

CHAPTER 7. RATIOS AND RATES

Available for free at Connexions <http://cnx.org/content/col10615/1.4>

Chapter 8

Techniques of Estimation

8.1 Objectives1

After completing this chapter, you should

Estimation by Rounding (Section 8.2)

• understand the reason for estimation

• be able to estimate the result of an addition, multiplication, subtraction, or division using the rounding technique

Estimation by Clustering (Section 8.3)

• understand the concept of clustering

• be able to estimate the result of adding more than two numbers when clustering occurs using the clustering technique

Mental ArithmeticUsing the Distributive Property (Section 8.4)

• understand the distributive property

• be able to obtain the exact result of a multiplication using the distributive property Estimation by Rounding Fractions (Section 8.5)

• be able to estimate the sum of two or more fractions using the technique of rounding fractions 8.2 Estimation by Rounding2

8.2.1 Section Overview

• Estimation By Rounding

When beginning a computation, it is valuable to have an idea of what value to expect for the result. When a computation is completed, it is valuable to know if the result is reasonable.

In the rounding process, it is important to note two facts:

1This content is available online at <http://cnx.org/content/m18896/1.3/>.

2This content is available online at <http://cnx.org/content/m35011/1.2/>.

Available for free at Connexions <http://cnx.org/content/col10615/1.4> 477

478

CHAPTER 8. TECHNIQUES OF ESTIMATION

1. The rounding that is done in estimation does not always follow the rules of rounding discussed in Section 1.4 (Rounding Whole Numbers). Since estimation is concerned with the expected value of a computation, rounding is done using convenience as the guide rather than using hard-and-fast rounding rules. For example, if we wish to estimate the result of the division 80 ÷ 26, we might round 26 to 20

rather than to 30 since 80 is more conveniently divided by 20 than by 30.

2. Since rounding may occur out of convenience, and dierent people have dierent ideas of what may be convenient, results of an estimation done by rounding may vary. For a particular computation, dierent people may get dierent estimated results. Results may vary.

Estimation

Estimation is the process of determining an expected value of a computation.

Common words used in estimation are about, near, and between.

8.2.2 Estimation by Rounding

The rounding technique estimates the result of a computation by rounding the numbers involved in the computation to one or two nonzero digits.

8.2.2.1 Sample Set A

Example 8.1

Estimate the sum: 2,357 + 6,106.

Notice that 2,357 is near 2, 400, and that 6,106 is near 6, 100.

|

{z

}

|

{z

}

two nonzero

two nonzero

digits

digits

The sum can be estimated by 2,400 + 6,100 = 8,500. (It is quick and easy to add 24 and 61.) Thus, 2,357 + 6,106 is about 8,400. In fact, 2,357 + 6,106 = 8,463.

8.2.2.2 Practice Set A

Exercise 8.2.1

(Solution on p. 505.)

Estimate the sum: 4,216 + 3,942.

Exercise 8.2.2

(Solution on p. 505.)

Estimate the sum: 812 + 514.

Exercise 8.2.3

(Solution on p. 505.)

Estimate the sum: 43,892 + 92,106.

8.2.2.3 Sample Set B

Example 8.2

Estimate the dierence: 5, 203 − 3, 015.

Notice that 5,203 is near 5, 200, and that 3,015 is near 3, 000.

|

{z

}

|

{z

}

two nonzero

one nonzero

digits

digit

The dierence can be estimated by 5, 200 − 3, 000 = 2,200.

Available for free at Connexions <http://cnx.org/content/col10615/1.4>

479

Thus, 5, 203 − 3, 015 is about 2,200. In fact, 5, 203 − 3, 015 = 2,188.

We could make a less accurate estimation by observing that 5,203 is near 5,000. The number 5,000

has only one nonzero digit rather than two (as does 5,200). This fact makes the estimation quicker (but a little less accurate). We then estimate the dierence by 5, 000−3, 000 = 2,000, and conclude that 5, 203 − 3, 015 is about 2,000. This is why we say “answers may vary.”

8.2.2.4 Practice Set B

Exercise 8.2.4

(Solution on p. 505.)

Estimate the dierence: 628 − 413.

Exercise 8.2.5

(Solution on p. 505.)

Estimate the dierence: 7, 842 − 5, 209.

Exercise 8.2.6

(Solution on p. 505.)

Estimate the dierence: 73, 812 − 28, 492.

8.2.2.5 Sample Set C

Example 8.3

Estimate the product: 73 · 46.

Notice that 73 is near

70,

and that 46 is near

50.

|{z}

|{z}

one nonzero

one nonzero

digit

digit

The product can be estimated by 70 · 50 = 3,500. (Recall that to multiply numbers ending in zeros, we multiply the nonzero digits and ax to this product the total number of ending zeros in the factors. See Section 2.2 for a review of this technique.)

Thus, 73 · 46 is about 3,500. In fact, 73 · 46 = 3,358.

Example 8.4

Estimate the product: 87 · 4,316.

Notice that 87 is close to

90,

and that 4,316 is close to 4, 000.

|{z}

|

{z

}

one nonzero

one nonzero

digit

digit

The product can be estimated by 90 · 4,000 = 360,000.

Thus, 87 · 4,316 is about 360,000. In fact, 87 · 4,316 = 375,492.

8.2.2.6 Practice Set C

Exercise 8.2.7

(Solution on p. 505.)

Estimate the product: 31 · 87.

Exercise 8.2.8

(Solution on p. 505.)

Estimate the product: 18 · 42.

Exercise 8.2.9

(Solution on p. 505.)

Estimate the product: 16 · 94.

Available for free at Connexions <http://cnx.org/content/col10615/1.4>

480

CHAPTER 8. TECHNIQUES OF ESTIMATION

8.2.2.7 Sample Set D

Example 8.5

Estimate the quotient: 153 ÷ 17.

Notice that 153 is close to

150,

and that 17 is close to

15.

|{z}

|{z}

two nonzero

two nonzero

digits

digits

The quotient can be estimated by 150 ÷ 15 = 10.

Thus, 153 ÷ 17 is about 10. In fact, 153 ÷ 17 = 9.

Example 8.6

Estimate the quotient: 742,000 ÷ 2,400.

Notice that 742,000 is close to 700, 000 , and that 2,400 is close to 2, 000.

|

{z

}

|

{z

}

one nonzero

one nonzero

digit

digit

The quotient can be estimated by 700,000 ÷ 2,000 = 350.

Thus, 742,000 ÷ 2,400 is about 350. In fact, 742,000 ÷ 2,400 = 309.16.

8.2.2.8 Practice Set D

Exercise 8.2.10

(Solution on p. 505.)

Estimate the quotient: 221 ÷ 18.

Exercise 8.2.11

(Solution on p. 505.)

Estimate the quotient: 4,079 ÷ 381.

Exercise 8.2.12

(Solution on p. 505.)

Estimate the quotient: 609,000 ÷ 16,000.

8.2.2.9 Sample Set E

Example 8.7

Estimate the sum: 53.82 + 41.6.

Notice that 53.82 is close to

54,

and that 41.6 is close to

42.

|{z}

|{z}

two nonzero

two nonzero

digits

digits

The sum can be estimated by 54 + 42 = 96.

Thus, 53.82 + 41.6 is about 96. In fact, 53.82 + 41.6 = 95.42.

Available for free at Connexions <http://cnx.org/content/col10615/1.4>

481

8.2.2.10 Practice Set E

Exercise 8.2.13

(Solution on p. 505.)

Estimate the sum: 61.02 + 26.8.

Exercise 8.2.14

(Solution on p. 505.)

Estimate the sum: 109.12 + 137.88.

8.2.2.11 Sample Set F

Example 8.8

Estimate the product: (31.28) (14.2).

Notice that 31.28 is close to

30,

and that 14.2 is close to

15.

|{z}

|{z}

one nonzero

two nonzero

digit

digits

The product can be estimated by 30 · 15 = 450. ( 3 · 15 = 45, then ax one zero.) Thus, (31.28) (14.2) is about 450. In fact, (31.28) (14.2) = 444.176.

Example 8.9

Estimate 21% of 5.42.

Notice that 21% = .21 as a decimal, and that .21 is close to

.2.

|{z}

one nonzero

digit

Notice also that 5.42 is close to

5.

|{z}

one nonzero

digit

Then, 21% of 5.42 can be estimated by (.2) (5) = 1.

Thus, 21% of 5.42 is about 1. In fact, 21% of 5.42 is 1.1382.

8.2.2.12 Practice Set F

Exercise 8.2.15

(Solution on p. 505.)

Estimate the product: (47.8) (21.1).

Exercise 8.2.16

(Solution on p. 505.)

Estimate 32% of 14.88.

8.2.3 Exercises

Estimate each calculation using the method of rounding. After you have made an estimate, nd the exact value and compare this to the estimated result to see if your estimated value is reasonable. Results may vary.

Exercise 8.2.17

(Solution on p. 505.)

1, 402 + 2, 198

Exercise 8.2.18

3,481 + 4,216

Available for free at Connexions <http://cnx.org/content/col10615/1.4>

482

CHAPTER 8. TECHNIQUES OF ESTIMATION

Exercise 8.2.19

(Solution on p. 505.)

921 + 796

Exercise 8.2.20

611 + 806

Exercise 8.2.21

(Solution on p. 505.)

4, 681 + 9, 325

Exercise 8.2.22

6, 476 + 7, 814

Exercise 8.2.23

(Solution on p. 505.)

7, 805 − 4, 266

Exercise 8.2.24

8, 427 − 5, 342

Exercise 8.2.25

(Solution on p. 505.)

14, 106 − 8, 412

Exercise 8.2.26

26, 486 − 18, 931

Exercise 8.2.27

(Solution on p. 505.)

32 · 53

Exercise 8.2.28

67 · 42

Exercise 8.2.29

(Solution on p. 505.)

628 · 891

Exercise 8.2.30

426 · 741

Exercise 8.2.31

(Solution on p. 505.)

18, 012 · 32, 416

Exercise 8.2.32

22, 481 · 51, 076

Exercise 8.2.33

(Solution on p. 505.)

287 ÷ 19

Exercise 8.2.34

884 ÷ 33

Exercise 8.2.35

(Solution on p. 506.)

1, 254 ÷ 57

Exercise 8.2.36

2, 189 ÷ 42

Exercise 8.2.37

(Solution on p. 506.)

8, 092 ÷ 239

Exercise 8.2.38

2, 688 ÷ 48

Exercise 8.2.39

(Solution on p. 506.)

72.14 + 21.08

Exercise 8.2.40

43.016 + 47.58

Exercise 8.2.41

(Solution on p. 506.)

96.53 − 26.91

Available for free at Connexions <http://cnx.org/content/col10615/1.4>

483

Exercise 8.2.42

115.0012 − 25.018

Exercise 8.2.43

(Solution on p. 506.)

206.19 + 142.38

Exercise 8.2.44

592.131 + 211.6

Exercise 8.2.45

(Solution on p. 506.)

(32.12) (48.7)

Exercise 8.2.46

(87.013) (21.07)

Exercise 8.2.47

(Solution on p. 506.)

(3.003) (16.52)

Exercise 8.2.48

(6.032) (14.091)

Exercise 8.2.49

(Solution on p. 506.)

(114.06) (384.3)

Exercise 8.2.50

(5, 137.118) (263.56)

Exercise 8.2.51

(Solution on p. 506.)

(6.92) (0.88)

Exercise 8.2.52

(83.04) (1.03)

Exercise 8.2.53

(Solution on p. 506.)

(17.31) (.003)

Exercise 8.2.54

(14.016) (.016)

Exercise 8.2.55

(Solution on p. 506.)

93% of 7.01

Exercise 8.2.56

107% of 12.6

Exercise 8.2.57

(Solution on p. 506.)

32% of 15.3

Exercise 8.2.58

74% of 21.93

Exercise 8.2.59

(Solution on p. 506.)

18% of 4.118

Exercise 8.2.60

4% of .863

Exercise 8.2.61

(Solution on p. 506.)

2% of .0039

Available for free at Connexions <http://cnx.org/content/col10615/1.4>

Image 331

484

CHAPTER 8. TECHNIQUES OF ESTIMATION

8.2.3.1 Exercises for Review

Exercise 8.2.62

(Section 5.3) Find the dierence: 710 − 516.

Exercise 8.2.63

(Solution on p. 506.)

(Section 5.6) Find the value 6−14 .

6+ 1

4

Exercise 8.2.64

(Section 6.3) Convert the complex decimal 1.111 to a decimal.

4

Exercise 8.2.65

(Solution on p. 506.)

(Section 7.4) A woman 5 foot tall casts an 8-foot shadow at a particular time of the day. How tall is a tree that casts a 96-foot shadow at the same time of the day?

Exercise 8.2.66

(Section 7.7) 11.62 is 83% of what number?

8.3 Estimation by Clustering3

8.3.1 Section Overview

• Estimation by Clustering

Cluster

When more than two numbers are to be added, the sum may be estimated using the clustering technique.

The rounding technique could also be used, but if several of the numbers are seen to cluster (are seen to be close to) one particular number, the clustering technique provides a quicker estimate. Consider a sum such as

32 + 68 + 29 + 73

Notice two things:

1. There are more than two numbers to be added.

2. Clustering occurs.

(a) Both 68 and 73 cluster around 70, so 68 + 73 is close to 80 + 70 = 2 (70) = 140.

(b) Both 32 and 29 cluster around 30, so 32 + 29 is close to 30 + 30 = 2 (30) = 60.

The sum may be estimated by

(2 · 30) + (2 · 70) = 6 + 140

=

200

In fact, 32 + 68 + 29 + 73 = 202.

3This content is available online at <http://cnx.org/content/m35012/1.2/>.

Available for free at Connexions <http://cnx.org/content/col10615/1.4>

485

8.3.1.1 Sample Set A

Estimate each sum. Results may vary.

Example 8.10

27 + 48 + 31 + 52.

27 and 31 cluster near 30. Their sum is about 2 · 30 = 60.

48 and 52 cluster near 50. Their sum is about 2 · 50 = 100.

(2 · 30) + (2 · 50)

=

60 + 100

Thus, 27 + 48 + 31 + 52 is about

=

160

In fact, 27 + 48 + 31 + 52 = 158.

Example 8.11

88 + 21 + 19 + 91.

88 and 91 cluster near 90. Their sum is about 2 · 90 = 180.

21 and 19 cluster near 20. Their sum is about 2 · 20 = 40.

(2 · 90) + (2 · 20)

=

180 + 40

Thus, 88 + 21 + 19 + 91 is about

=

220

In fact, 88 + 21 + 19 + 91 = 219.

Example 8.12

17 + 21 + 48 + 18.

17, 21, and 18 cluster near 20. Their sum is about 3 · 20 = 60.

48 is about 50.

(3 · 20) + 50

=

60 + 50

Thus, 17 + 21 + 48 + 18 is about

=

110

In fact, 17 + 21 + 48 + 18 = 104.

Example 8.13

61 + 48 + 49 + 57 + 52.

61 and 57 cluster near 60. Their sum is about 2 · 60 = 120.

48, 49, and 52 cluster near 50. Their sum is about 3 · 50 = 150.

(2 · 60) + (3 · 50)

=

120 + 150

Thus, 61 + 48 + 49 + 57 + 52 is about

=

270

In fact, 61 + 48 + 49 + 57 + 52 = 267.

Example 8.14

706 + 321 + 293 + 684.

706 and 684 cluster near 700. Their sum is about 2 · 700 = 1,400.

Available for free at Connexions <http://cnx.org/content/col10615/1.4>

486

CHAPTER 8. TECHNIQUES OF ESTIMATION

321 and 293 cluster near 300. Their sum is about 2 · 300 = 600.

(2 · 700) + (2 · 300)

=

1, 400 + 600

Thus, 706 + 321 + 293 + 684 is about

=

2, 000

In fact, 706 + 321 + 293 + 684 = 2,004.

8.3.1.2 Practice Set A

Use the clustering method to estimate each sum.

Exercise 8.3.1

(Solution on p. 506.)

28 + 51 + 31 + 47

Exercise 8.3.2

(Solution on p. 506.)

42 + 39 + 68 + 41

Exercise 8.3.3

(Solution on p. 506.)

37 + 39 + 83 + 42 + 79

Exercise 8.3.4

(Solution on p. 506.)

612 + 585 + 830 + 794

8.3.2 Exercises

Use the clustering method to estimate each sum. Results may vary.

Exercise 8.3.5

(Solution on p. 506.)

28 + 51 + 31 + 47

Exercise 8.3.6

42 + 19 + 39 + 23

Exercise 8.3.7

(Solution on p. 506.)

88 + 62 + 59 + 90

Exercise 8.3.8

76 + 29 + 33 + 82

Exercise 8.3.9

(Solution on p. 506.)

19 + 23 + 87 + 21

Exercise 8.3.10

41 + 28 + 42 + 37

Exercise 8.3.11

(Solution on p. 506.)

89 + 32 + 89 + 93

Exercise 8.3.12

73 + 72 + 27 + 71

Exercise 8.3.13

(Solution on p. 506.)

43 + 62 + 61 + 55

Exercise 8.3.14

31 + 77 + 31 + 27

Exercise 8.3.15

(Solution on p. 507.)

57 + 34 + 28 + 61 + 62

Available for free at Connexions <http://cnx.org/content/col10615/1.4>

487

Exercise 8.3.16

94 + 18 + 23 + 91 + 19

Exercise 8.3.17

(Solution on p. 507.)

103 + 72 + 66 + 97 + 99

Exercise 8.3.18

42 + 121 + 119 + 124 + 41

Exercise 8.3.19

(Solution on p. 507.)

19 + 24 + 87 + 23 + 91 + 93

Exercise 8.3.20

108 + 61 + 63 + 96 + 57 + 99

Exercise 8.3.21

(Solution on p. 507.)

518 + 721 + 493 + 689

Exercise 8.3.22

981 + 1208 + 1214 + 1006

Exercise 8.3.23

(Solution on p. 507.)

23 + 81 + 77 + 79 + 19 + 81

Exercise 8.3.24

94 + 68 + 66 + 101 + 106 + 71 + 110

8.3.2.1 Exercises for Review

Exercise 8.3.25

(Solution on p. 507.)

(Section 1.2) Specify all the digits greater than 6.

Exercise 8.3.26

(Section 4.5) Find the product: 2 · 9

3

14 · 7

12 .

Exercise 8.3.27

(Solution on p. 507.)

(Section 6.3) Convert 0.06 to a fraction.

Exercise 8.3.28

(Section 7.3) Write the proportion in fractional form: “5 is to 8 as 25 is to 40.”

Exercise 8.3.29

(Solution on p. 507.)

(Section 8.2) Estimate the sum using the method of rounding: 4,882 + 2,704.

8.4 Mental Arithmetic-Using the Distributive Property4

8.4.1 Section Overview

• The Distributive Property

• Estimation Using the Distributive Property

4This content is available online at <http://cnx.org/content/m35013/1.2/>.

Available for free at Connexions <http://cnx.org/content/col10615/1.4>

Image 332

488

CHAPTER 8. TECHNIQUES OF ESTIMATION

8.4.2 The Distributive Property

Distributive Property

The distributive property is a characteristic of numbers that involves both addition and multiplication.

It is used often in algebra, and we can use it now to obtain exact results for a multiplication.

Suppose we wish to compute 3 (2 + 5). We can proceed in either of two ways, one way which is known to us already (the order of operations), and a new way (the distributive property).

1. Compute 3 (2 + 5) using the order of operations.

3 (2 + 5)

Operate inside the parentheses rst: 2 + 5 = 7.

3 (2 + 5) = 3 · 7

Now multiply 3 and 7.

3 (2 + 5) = 3 · 7 = 21

Thus, 3 (2 + 5) = 21.

2. Compute 3 (2 + 5) using the distributive property.

We know that multiplication describes repeated addition. Thus,

3 (2 + 5)

=

2 + 5 + 2 + 5 + 2 + 5

|

{z

}

2 + 5 appears 3 times

=

2 + 2 + 2 + 5 + 5 + 5

(by the commutative property of addition)

=

3 · 2 + 3 · 5

(since multiplication describes repeated addition)

=

6 + 15

=

21

Thus, 3 (2 + 5) = 21.

Let’s look again at this use of the distributive property.

3 (2 + 5)

=

2 + 5 + 2 + 5 + 2 + 5

|

{z

}

2 + 5 appears 3 times

3 (2 + 5)

=

2 + 2 + 2

+

5 + 5 + 5

|

{z

}

|

{z

}

2 appears 3 times

5 appears 3 times

The 3 has been distributed to the 2 and 5.

This is the distributive property. We distribute the factor to each addend in the parentheses. The distributive property works for both sums and dierences.

Available for free at Connexions <http://cnx.org/content/col10615/1.4>

Image 333

Image 334

Image 335

Image 336

489

8.4.2.1 Sample Set A

Example 8.15

Using the order of operations, we get

4 (6 + 2)

=

4 · 8

=

32

Example 8.16

Using the order of operations, we get

8 (9 + 6)

=

8 · 15

=

120

Example 8.17

Example 8.18

8.4.2.2 Practice Set A

Use the distributive property to compute each value.

Exercise 8.4.1

(Solution on p. 507.)

6 (8 + 4)

Exercise 8.4.2

(Solution on p. 507.)

4 (4 + 7)

Exercise 8.4.3

(Solution on p. 507.)

8 (2 + 9)

Exercise 8.4.4

(Solution on p. 507.)

12 (10 + 3)

Exercise 8.4.5

(Solution on p. 507.)

6 (11 − 3)

Exercise 8.4.6

(Solution on p. 507.)

8 (9 − 7)

Exercise 8.4.7

(Solution on p. 507.)

15 (30 − 8)

Available for free at Connexions <http://cnx.org/content/col10615/1.4>

Image 337

Image 338

Image 339

Image 340

490

CHAPTER 8. TECHNIQUES OF ESTIMATION

8.4.3 Estimation Using the Distributive Property

We can use the distributive property to obtain exact results for products such as 25 · 23. The distributive property works best for products when one of the factors ends in 0 or 5. We shall restrict our attention to only such products.

8.4.3.1 Sample Set B

Use the distributive property to compute each value.

Example 8.19

25 · 23

Notice that 23 = 20 + 3. We now write

Thus, 25 · 23 = 575

We could have proceeded by writing 23 as 30 − 7.

Example 8.20

15 · 37

Notice that 37 = 30 + 7. We now write

Thus, 15 · 37 = 555

We could have proceeded by writing 37 as 40 − 3.

Example 8.21

15 · 86

Notice that 86 = 80 + 6. We now write

Available for free at Connexions <http://cnx.org/content/col10615/1.4>

Image 341

Image 342

491

We could have proceeded by writing 86 as 90 − 4.

8.4.3.2 Practice Set B

Use the distributive property to compute each value.

Exercise 8.4.8

(Solution on p. 507.)

25 · 12

Exercise 8.4.9

(Solution on p. 507.)

35 · 14

Exercise 8.4.10

(Solution on p. 507.)

80 · 58

Exercise 8.4.11

(Solution on p. 507.)

65 · 62

8.4.4 Exercises

Use the distributive property to compute each product.

Exercise 8.4.12

(Solution on p. 507.)

15 · 13

Exercise 8.4.13

15 · 14

Exercise 8.4.14

(Solution on p. 507.)

25 · 11

Exercise 8.4.15

25 · 16

Exercise 8.4.16

(Solution on p. 507.)

15 · 16

Exercise 8.4.17

35 · 12

Exercise 8.4.18

(Solution on p. 507.)

45 · 83

Exercise 8.4.19

45 · 38

Exercise 8.4.20

(Solution on p. 507.)

25 · 38

Available for free at Connexions <http://cnx.org/content/col10615/1.4>

492

CHAPTER 8. TECHNIQUES OF ESTIMATION

Exercise 8.4.21

25 · 96

Exercise 8.4.22

(Solution on p. 507.)

75 · 14

Exercise 8.4.23

85 · 34

Exercise 8.4.24

(Solution on p. 508.)

65 · 26

Exercise 8.4.25

55 · 51

Exercise 8.4.26

(Solution on p. 508.)

15 · 107

Exercise 8.4.27

25 · 208

Exercise 8.4.28

(Solution on p. 508.)

35 · 402

Exercise 8.4.29

85 · 110

Exercise 8.4.30

(Solution on p. 508.)

95 · 12

Exercise 8.4.31

65 · 40

Exercise 8.4.32

(Solution on p. 508.)

80 · 32

Exercise 8.4.33

30 · 47

Exercise 8.4.34

(Solution on p. 508.)

50 · 63

Exercise 8.4.35

90 · 78

Exercise 8.4.36

(Solution on p. 508.)

40 · 89

8.4.4.1 Exercises for Review

Exercise 8.4.37

(Section 3.5) Find the greatest common factor of 360 and 3,780.

Exercise 8.4.38

(Solution on p. 508.)

(Section 4.5) Reduce 594

5,148 to lowest terms.

Exercise 8.4.39

(Section 4.7) 15 of 24 is what number?

9

7

Exercise 8.4.40

(Solution on p. 508.)

(Section 7.3) Solve the proportion: 715 = x90.

Exercise 8.4.41

(Section 8.3) Use the clustering method to estimate the sum: 88 + 106 + 91 + 114.

Available for free at Connexions <http://cnx.org/content/col10615/1.4>

493

8.5 Estimation by Rounding Fractions5

8.5.1 Section Overview

• Estimation by Rounding Fractions

Estimation by rounding fractions is a useful technique for estimating the result of a computation involving fractions. Fractions are commonly rounded to 1, 1, 3, 0, and 1. Remember that rounding may cause 4

2

4

estimates to vary.

8.5.1.1 Sample Set A

Make each estimate remembering that results may vary.

Example 8.22

Estimate 3 + 5

5

12 .

Notice that 3 is about 1, and that 5

.

5

2

12 is about 12

Thus, 3 + 5

+ 1 = 1. In fact, 3 + 5

5

12 is about 12

2

5

12 = 61

60 , a little more than 1.

Example 8.23

Estimate 53 + 4 9

.

8

10 + 11 15

Adding the whole number parts, we get 20. Notice that 3 is close to 1, 9

is

8

4

10 is close to 1, and 15

close to 1. Then 3 + 9

is close to 1 + 1 + 1 = 11.

4

8

10 + 15

4

4

2

Thus, 53 + 4 9

is close to 20 + 11 = 211.

8

10 + 11 15

2

2

In fact, 53 + 4 9

= 21 19

.

8

10 + 11 15

40 , a little less than 21 12

8.5.1.2 Practice Set A

Use the method of rounding fractions to estimate the result of each computation. Results may vary.

Exercise 8.5.1

(Solution on p. 508.)

5 + 5

8

12

Exercise 8.5.2

(Solution on p. 508.)

7 + 3

9

5

Exercise 8.5.3

(Solution on p. 508.)

8 4

15 + 3 7

10

Exercise 8.5.4

(Solution on p. 508.)

16 1 + 47

20

8

5This content is available online at <http://cnx.org/content/m35014/1.2/>.

Available for free at Connexions <http://cnx.org/content/col10615/1.4>

494

CHAPTER 8. TECHNIQUES OF ESTIMATION

8.5.2 Exercises

Estimate each sum or dierence using the method of rounding. After you have made an estimate, nd the exact value of the sum or dierence and compare this result to the estimated value. Result may vary.

Exercise 8.5.5

(Solution on p. 508.)

5 + 7

6

8

Exercise 8.5.6

3 + 11

8

12

Exercise 8.5.7

(Solution on p. 508.)

9

10 + 35

Exercise 8.5.8

13

15 + 1

20

Exercise 8.5.9

(Solution on p. 508.)

3

20 + 6

25

Exercise 8.5.10

1

12 + 45

Exercise 8.5.11

(Solution on p. 508.)

15

16 + 1

12

Exercise 8.5.12

29

30 + 11

20

Exercise 8.5.13

(Solution on p. 508.)

5

12 + 6 4

11

Exercise 8.5.14

3 + 8 4

7

15

Exercise 8.5.15

(Solution on p. 508.)

9

10 + 2 38

Exercise 8.5.16

19

20 + 15 59

Exercise 8.5.17

(Solution on p. 508.)

8 3 + 4 1

5

20

Exercise 8.5.18

5 3

20 + 2 8

15

Exercise 8.5.19

(Solution on p. 508.)

9 1

15 + 6 45

Exercise 8.5.20

7 5

12 + 10 1

16

Exercise 8.5.21

(Solution on p. 508.)

3 11

20 + 2 13

25 + 1 78

Exercise 8.5.22

6 1

12 + 1 1

10 + 5 56

Exercise 8.5.23

(Solution on p. 508.)

15

16 − 78

Exercise 8.5.24

12

25 − 9

20

Available for free at Connexions <http://cnx.org/content/col10615/1.4>

495

8.5.2.1 Exercises for Review

Exercise 8.5.25

(Solution on p. 508.)

(Section 2.6) The fact that

(a rst number · a second number)

·

a third number

=

a rst number

·

(a second number · a third number)

is an example of which property of multiplication?

Exercise 8.5.26

(Section 4.6) Find the quotient: 14

15 ÷ 4

45 .

Exercise 8.5.27

(Solution on p. 508.)

(Section 5.4) Find the dierence: 35 − 22.

9

3

Exercise 8.5.28

(Section 6.8) Find the quotient: 4.6 ÷ 0.11.

Exercise 8.5.29

(Solution on p. 509.)

(Section 8.4) Use the distributive property to compute the product: 25 · 37.

Available for free at Connexions <http://cnx.org/content/col10615/1.4>

496

CHAPTER 8. TECHNIQUES OF ESTIMATION

8.6 Summary of Key Concepts6

8.6.1 Summary of Key Concepts

Estimation (Section 8.2)

Estimation is the process of determining an expected value of a computation.

Estimation By Rounding (Section 8.2)

The rounding technique estimates the result of a computation by rounding the numbers involved in the computation to one or two nonzero digits. For example, 512 + 896 can be estimated by 500 + 900 = 1,400.

Cluster (Section 8.3)

When several numbers are close to one particular number, they are said to cluster near that particular number.

Estimation By Clustering (Section 8.3)

The clustering technique of estimation can be used when

1. there are more than two numbers to be added, and

2. clustering occurs.

For example, 31 + 62 + 28 + 59 can be estimated by

(2 · 30) + (2 · 60) = 60 + 120 = 180

Distributive Property (Section 8.4)

The distributive property is a characteristic of numbers that involves both addition and multiplication.

For example,

3 (4 + 6) = 3 · 4 + 3 · 6 = 12 + 18 = 30

Estimation Using the Distributive Property (Section 8.4)

The distributive property can be used to obtain exact results for a multiplication.

For example,

15 · 23 = 15 · (20 + 3) = 15 · 20 + 15 · 3 = 300 + 45 = 345

Estimation by Rounding Fractions (Section 8.5)

Estimation by rounding fractions commonly rounds fractions to 1, 1, 3, 0, and 1.

4

2

4

For example,

5

12 + 5

16 can be estimated by 1 + 1 = 3

2

4

4

8.7 Exercise Supplement7

8.7.1 Exercise Supplement

8.7.1.1 Estimation by Rounding (Section 8.2)

For problems 1-70, estimate each value using the method of rounding. After you have made an estimate, nd the exact value. Compare the exact and estimated values. Results may vary.

Exercise 8.7.1

(Solution on p. 509.)

286 + 312

Exercise 8.7.2

419 + 582

6This content is available online at <http://cnx.org/content/m35015/1.2/>.

7This content is available online at <http://cnx.org/content/m35016/1.2/>.

Available for free at Connexions <http://cnx.org/content/col10615/1.4>

497

Exercise 8.7.3

(Solution on p. 509.)

689 + 511

Exercise 8.7.4

926 + 1, 105

Exercise 8.7.5

(Solution on p. 509.)

1, 927 + 3, 017

Exercise 8.7.6

5, 026 + 2, 814

Exercise 8.7.7

(Solution on p. 509.)

1, 408 + 2, 352

Exercise 8.7.8

1, 186 + 4, 228

Exercise 8.7.9

(Solution on p. 509.)

5, 771 + 246

Exercise 8.7.10

8, 305 + 484

Exercise 8.7.11

(Solution on p. 509.)

3, 812 + 2, 906

Exercise 8.7.12

5, 293 + 8, 007

Exercise 8.7.13

(Solution on p. 509.)

28, 481 + 32, 856

Exercise 8.7.14

92, 512 + 26, 071

Exercise 8.7.15

(Solution on p. 509.)

87, 612 + 2, 106

Exercise 8.7.16

42, 612 + 4, 861

Exercise 8.7.17

(Solution on p. 509.)

212, 413 + 609

Exercise 8.7.18

487, 235 + 494

Exercise 8.7.19

(Solution on p. 509.)

2, 409 + 1, 526

Exercise 8.7.20

3, 704 + 4, 704

Exercise 8.7.21

(Solution on p. 509.)

41 · 63

Exercise 8.7.22

38 · 81

Exercise 8.7.23

(Solution on p. 509.)

18 · 28

Exercise 8.7.24

52 · 21

Exercise 8.7.25

(Solution on p. 509.)

307 · 489

Available for free at Connexions <http://cnx.org/content/col10615/1.4>

498

CHAPTER 8. TECHNIQUES OF ESTIMATION

Exercise 8.7.26

412 · 807

Exercise 8.7.27

(Solution on p. 509.)

77 · 614

Exercise 8.7.28

62 · 596

Exercise 8.7.29

(Solution on p. 509.)

27 · 473

Exercise 8.7.30

92 · 336

Exercise 8.7.31

(Solution on p. 509.)

12 · 814

Exercise 8.7.32

8 · 2, 106

Exercise 8.7.33

(Solution on p. 509.)

192 · 452

Exercise 8.7.34

374 · 816

Exercise 8.7.35

(Solution on p. 509.)

88 · 4, 392

Exercise 8.7.36

126 · 2, 834

Exercise 8.7.37

(Solution on p. 509.)

3, 896 · 413

Exercise 8.7.38

5, 794 · 837

Exercise 8.7.39

(Solution on p. 509.)

6, 311 · 3, 512

Exercise 8.7.40

7, 471 · 5, 782

Exercise 8.7.41

(Solution on p. 509.)

180 ÷ 12

Exercise 8.7.42

309 ÷ 16

Exercise 8.7.43

(Solution on p. 509.)

286 ÷ 22

Exercise 8.7.44

527 ÷ 17

Exercise 8.7.45

(Solution on p. 509.)

1, 007 ÷ 19

Exercise 8.7.46

1, 728 ÷ 36

Exercise 8.7.47

(Solution on p. 509.)

2, 703 ÷ 53

Exercise 8.7.48

2, 562 ÷ 61

Available for free at Connexions <http://cnx.org/content/col10615/1.4>

499

Exercise 8.7.49

(Solution on p. 510.)

1, 260 ÷ 12

Exercise 8.7.50

3, 618 ÷ 18

Exercise 8.7.51

(Solution on p. 510.)

3, 344 ÷ 76

Exercise 8.7.52

7, 476 ÷ 356

Exercise 8.7.53

(Solution on p. 510.)

20, 984 ÷ 488

Exercise 8.7.54

43, 776 ÷ 608

Exercise 8.7.55

(Solution on p. 510.)

7, 196 ÷ 514

Exercise 8.7.56

51, 492 ÷ 514

Exercise 8.7.57

(Solution on p. 510.)

26, 962 ÷ 442

Exercise 8.7.58

33, 712 ÷ 112

Exercise 8.7.59

(Solution on p. 510.)

105, 152 ÷ 106

Exercise 8.7.60

176, 978 ÷ 214

Exercise 8.7.61

(Solution on p. 510.)

48.06 + 23.11

Exercise 8.7.62

73.73 + 72.9

Exercise 8.7.63

(Solution on p. 510.)

62.91 + 56.4

Exercise 8.7.64

87.865 + 46.772

Exercise 8.7.65

(Solution on p. 510.)

174.6 + 97.2

Exercise 8.7.66

(48.3) (29.6)

Exercise 8.7.67

(Solution on p. 510.)

(87.11) (23.2)

Exercise 8.7.68

(107.02) (48.7)

Exercise 8.7.69

(Solution on p. 510.)

(0.76) (5.21)

Exercise 8.7.70

(1.07) (13.89)

Available for free at Connexions <http://cnx.org/content/col10615/1.4>

500

CHAPTER 8. TECHNIQUES OF ESTIMATION

8.7.1.2 Estimation by Clustering (Section 8.3)

For problems 71-90, estimate each value using the method of clustering. After you have made an estimate, nd the exact value. Compare the exact and estimated values. Results may vary.

Exercise 8.7.71

(Solution on p. 510.)

38 + 51 + 41 + 48

Exercise 8.7.72

19 + 73 + 23 + 71

Exercise 8.7.73

(Solution on p. 510.)

27 + 62 + 59 + 31

Exercise 8.7.74

18 + 73 + 69 + 19

Exercise 8.7.75

(Solution on p. 510.)

83 + 49 + 79 + 52

Exercise 8.7.76

67 + 71 + 84 + 81

Exercise 8.7.77

(Solution on p. 510.)

16 + 13 + 24 + 26

Exercise 8.7.78

34 + 56 + 36 + 55

Exercise 8.7.79

(Solution on p. 510.)

14 + 17 + 83 + 87

Exercise 8.7.80

93 + 108 + 96 + 111

Exercise 8.7.81

(Solution on p. 510.)

18 + 20 + 31 + 29 + 24 + 38

Exercise 8.7.82

32 + 27 + 48 + 51 + 72 + 69

Exercise 8.7.83

(Solution on p. 510.)

64 + 17 + 27 + 59 + 31 + 21

Exercise 8.7.84

81 + 41 + 92 + 38 + 88 + 80

Exercise 8.7.85

(Solution on p. 510.)

87 + 22 + 91

Exercise 8.7.86

44 + 38 + 87

Exercise 8.7.87

(Solution on p. 510.)

19 + 18 + 39 + 22 + 42

Exercise 8.7.88

31 + 28 + 49 + 29

Exercise 8.7.89

(Solution on p. 510.)

88 + 86 + 27 + 91 + 29

Exercise 8.7.90

57 + 62 + 18 + 23 + 61 + 21

Available for free at Connexions <http://cnx.org/content/col10615/1.4>

501

8.7.1.3 Mental Arithmetic- Using the Distributive Property (Section 8.4) For problems 91-110, compute each product using the distributive property.

Exercise 8.7.91

(Solution on p. 510.)

15 · 33

Exercise 8.7.92

15 · 42

Exercise 8.7.93

(Solution on p. 510.)

35 · 36

Exercise 8.7.94

35 · 28

Exercise 8.7.95

(Solution on p. 510.)

85 · 23

Exercise 8.7.96

95 · 11

Exercise 8.7.97

(Solution on p. 510.)

30 · 14

Exercise 8.7.98

60 · 18

Exercise 8.7.99

(Solution on p. 511.)

75 · 23

Exercise 8.7.100

65 · 31

Exercise 8.7.101

(Solution on p. 511.)

17 · 15

Exercise 8.7.102

38 · 25

Exercise 8.7.103

(Solution on p. 511.)

14 · 65

Exercise 8.7.104

19 · 85

Exercise 8.7.105

(Solution on p. 511.)

42 · 60

Exercise 8.7.106

81 · 40

Exercise 8.7.107

(Solution on p. 511.)

15 · 105

Exercise 8.7.108

35 · 202

Exercise 8.7.109

(Solution on p. 511.)

45 · 306

Exercise 8.7.110

85 · 97

Available for free at Connexions <http://cnx.org/content/col10615/1.4>

502

CHAPTER 8. TECHNIQUES OF ESTIMATION

8.7.1.4 Estimation by Rounding Fractions (Section 8.5)

For problems 111-125, estimate each sum using the method of rounding fractions. After you have made an estimate, nd the exact value. Compare the exact and estimated values. Results may vary.

Exercise 8.7.111

(Solution on p. 511.)

3 + 5

8

6

Exercise 8.7.112

7

16 + 1

24

Exercise 8.7.113

(Solution on p. 511.)

7

15 + 13

30

Exercise 8.7.114

14

15 + 19

20

Exercise 8.7.115

(Solution on p. 511.)

13

25 + 7

30

Exercise 8.7.116

11

12 + 78

Exercise 8.7.117

(Solution on p. 511.)

9

32 + 15

16

Exercise 8.7.118

5 + 1

8

32

Exercise 8.7.119

(Solution on p. 511.)

2 3 + 6 3

4

5

Exercise 8.7.120

4 5 + 8 1

9

27

Exercise 8.7.121

(Solution on p. 511.)

11 518 + 722

45

Exercise 8.7.122

1419

36 + 2 7

18

Exercise 8.7.123

(Solution on p. 511.)

6 1

20 + 2 1

10 + 8 13

60

Exercise 8.7.124

5 7 + 1 1 + 12 5

8

4

12

Exercise 8.7.125

(Solution on p. 511.)

101 + 615

2

16 + 8 19

80

8.8 Prociency Exam8

8.8.1 Prociency Exam

For problems 1 – 16, estimate each value. After you have made an estimate, nd the exact value. Results may vary.

Exercise 8.8.1

(Solution on p. 511.)

(Section 8.2) 3,716 + 6,789

Exercise 8.8.2

(Solution on p. 511.)

(Section 8.2) 8,821 + 9,217

8This content is available online at <http://cnx.org/content/m35017/1.2/>.

Available for free at Connexions <http://cnx.org/content/col10615/1.4>

503

Exercise 8.8.3

(Solution on p. 511.)

(Section 8.2) 7, 316 − 2, 305

Exercise 8.8.4

(Solution on p. 511.)

(Section 8.2) 110, 812 − 83, 406

Exercise 8.8.5

(Solution on p. 511.)

(Section 8.2) 82 · 38

Exercise 8.8.6

(Solution on p. 511.)

(Section 8.2) 51 · 92

Exercise 8.8.7

(Solution on p. 511.)

(Section 8.2) 48 · 6,012

Exercise 8.8.8

(Solution on p. 511.)

(Section 8.2) 238 ÷ 17

Exercise 8.8.9

(Solution on p. 511.)

(Section 8.2) 2,660 ÷ 28

Exercise 8.8.10

(Solution on p. 511.)

(Section 8.2) 43.06 + 37.94

Exercise 8.8.11

(Solution on p. 511.)

(Section 8.2) 307.006 + 198.0005

Exercise 8.8.12

(Solution on p. 512.)

(Section 8.2) (47.2) (92.8)

Exercise 8.8.13

(Solution on p. 512.)

(Section 8.3) 58 + 91 + 61 + 88

Exercise 8.8.14

(Solution on p. 512.)

(Section 8.3) 43 + 39 + 89 + 92

Exercise 8.8.15

(Solution on p. 512.)

(Section 8.3) 81 + 78 + 27 + 79

Exercise 8.8.16

(Solution on p. 512.)

(Section 8.3) 804 + 612 + 801 + 795 + 606

For problems 17-21, use the distributive property to obtain the exact result.

Exercise 8.8.17

(Solution on p. 512.)

(Section 8.4) 25 · 14

Exercise 8.8.18

(Solution on p. 512.)

(Section 8.4) 15 · 83

Exercise 8.8.19

(Solution on p. 512.)

(Section 8.4) 65 · 98

Exercise 8.8.20

(Solution on p. 512.)

(Section 8.4) 80 · 107

Exercise 8.8.21

(Solution on p. 512.)

(Section 8.4) 400 · 215

For problems 22-25, estimate each value. After you have made an estimate, nd the exact value. Results may vary.

Exercise 8.8.22

(Solution on p. 512.)

(Section 8.5) 15

16 + 58

Exercise 8.8.23

(Solution on p. 512.)

(Section 8.5 ) 125 + 11

20 + 17

30

Available for free at Connexions <http://cnx.org/content/col10615/1.4>

504

CHAPTER 8. TECHNIQUES OF ESTIMATION

Exercise 8.8.24

(Solution on p. 512.)

(Section 8.5) 8 916 + 14 112

Exercise 8.8.25

(Solution on p. 512.)

(Section 8.5) 54 + 117

9

36 + 6 5

12

Available for free at Connexions <http://cnx.org/content/col10615/1.4>

505

Solutions to Exercises in Chapter 8

Solution to Exercise 8.2.1 (p. 478)

4, 216 + 3, 942 : 4, 200 + 3, 900. About 8,100. In fact, 8,158.

Solution to Exercise 8.2.2 (p. 478)

812 + 514 : 800 + 500. About 1,300. In fact, 1,326.

Solution to Exercise 8.2.3 (p. 478)

43, 892 + 92, 106 : 44, 000 + 92, 000. About 136,000. In fact, 135,998.

Solution to Exercise 8.2.4 (p. 479)

628 − 413 : 600 − 400. About 200. In fact, 215.

Solution to Exercise 8.2.5 (p. 479)

7, 842 − 5, 209 : 7, 800 − 5, 200. About 2,600. In fact, 2,633.

Solution to Exercise 8.2.6 (p. 479)

73, 812 − 28, 492 : 74, 000 − 28, 000. About 46,000. In fact, 45,320.

Solution to Exercise 8.2.7 (p. 479)

31 · 87 : 30 · 90. About 2,700. In fact, 2,697.

Solution to Exercise 8.2.8 (p. 479)

18 · 42 : 20 · 40. About 800. In fact, 756.

Solution to Exercise 8.2.9 (p. 479)

16 · 94 : 15 · 100. About 1,500. In fact, 1,504.

Solution to Exercise 8.2.10 (p. 480)

221 ÷ 18 : 200 ÷ 20. About 10. In fact, 12.27.

Solution to Exercise 8.2.11 (p. 480)

4, 079 ÷ 381 : 4, 000 ÷ 400. About 10. In fact, 10.70603675…

Solution to Exercise 8.2.12 (p. 480)

609, 000 ÷ 16, 000 : 600, 000 ÷ 15, 000. About 40. In fact, 38.0625.

Solution to Exercise 8.2.13 (p. 481)

61.02 + 26.8 : 61 + 27. About 88. In fact, 87.82.

Solution to Exercise 8.2.14 (p. 481)

109.12 + 137.88 : 110 + 138. About 248. In fact, 247. We could have estimated 137.88 with 140. Then 110 + 140 is an easy mental addition. We would conclude then that 109.12 + 137.88 is about 250.

Solution to Exercise 8.2.15 (p. 481)

(47.8) (21.1) : (50) (20). About 1,000. In fact, 1,008.58.

Solution to Exercise 8.2.16 (p. 481)

32% of 14.88 : (.3) (15). About 4.5. In fact, 4.7616.

Solution to Exercise 8.2.17 (p. 481)

about 3,600; in fact 3,600

Solution to Exercise 8.2.19 (p. 482)

about 1,700; in fact 1,717

Solution to Exercise 8.2.21 (p. 482)

about 14,000; in fact 14,006

Solution to Exercise 8.2.23 (p. 482)

about 3,500; in fact 3,539

Solution to Exercise 8.2.25 (p. 482)

about 5,700; in fact 5,694

Solution to Exercise 8.2.27 (p. 482)

about 1,500; in fact 1,696

Solution to Exercise 8.2.29 (p. 482)

about 540,000; in fact 559,548

Solution to Exercise 8.2.31 (p. 482)

about 583,200,000; in fact 583,876,992

Available for free at Connexions <http://cnx.org/content/col10615/1.4>

506

CHAPTER 8. TECHNIQUES OF ESTIMATION

Solution to Exercise 8.2.33 (p. 482)

about 15; in fact 15.11

Solution to Exercise 8.2.35 (p. 482)

about 20; in fact 22

Solution to Exercise 8.2.37 (p. 482)

about 33; in fact 33.86

Solution to Exercise 8.2.39 (p. 482)

about 93.2; in fact 93.22

Solution to Exercise 8.2.41 (p. 482)

about 70; in fact 69.62

Solution to Exercise 8.2.43 (p. 483)

about 348.6; in fact 348.57

Solution to Exercise 8.2.45 (p. 483)

about 1,568.0; in fact 1,564.244

Solution to Exercise 8.2.47 (p. 483)

about 49.5; in fact 49.60956

Solution to Exercise 8.2.49 (p. 483)

about 43,776; in fact 43,833.258

Solution to Exercise 8.2.51 (p. 483)

about 6.21; in fact 6.0896

Solution to Exercise 8.2.53 (p. 483)

about 0.0519; in fact 0.05193

Solution to Exercise 8.2.55 (p. 483)

about 6.3; in fact 6.5193

Solution to Exercise 8.2.57 (p. 483)

about 4.5; in fact 4.896

Solution to Exercise 8.2.59 (p. 483)

about 0.8; in fact 0.74124

Solution to Exercise 8.2.61 (p. 483)

about 0.00008; in fact 0.000078

Solution to Exercise 8.2.63 (p. 484)

23

25

Solution to Exercise 8.2.65 (p. 484)

60 feet tall

Solution to Exercise 8.3.1 (p. 486)

(2 · 30) + (2 · 50) = 60 + 100 = 160

Solution to Exercise 8.3.2 (p. 486)

(3 · 40) + 70 = 120 + 70 = 190

Solution to Exercise 8.3.3 (p. 486)

(3 · 40) + (2 · 80) = 120 + 160 = 280

Solution to Exercise 8.3.4 (p. 486)

(2 · 600) + (2 · 800) = 1, 200 + 1, 600 = 2, 800

Solution to Exercise 8.3.5 (p. 486)

2 (30) + 2 (50) = 160 (157)

Solution to Exercise 8.3.7 (p. 486)

2 (90) + 2 (60) = 300 (299)

Solution to Exercise 8.3.9 (p. 486)

3 (20) + 90 = 150 (150)

Solution to Exercise 8.3.11 (p. 486)

3 (90) + 30 = 300 (303)

Available for free at Connexions <http://cnx.org/content/col10615/1.4>

507

Solution to Exercise 8.3.13 (p. 486)

40 + 3 (60) = 220 (221)

Solution to Exercise 8.3.15 (p. 486)

3 (60) + 2 (30) = 240 (242)

Solution to Exercise 8.3.17 (p. 487)

3 (100) + 2 (70) = 440 (437)

Solution to Exercise 8.3.19 (p. 487)

3 (20) + 3 (90) = 330 (337)

Solution to Exercise 8.3.21 (p. 487)

2 (500) + 2 (700) = 2, 400 (2, 421)

Solution to Exercise 8.3.23 (p. 487)

2 (20) + 4 (80) = 360 (360)

Solution to Exercise 8.3.25 (p. 487)

7, 8, 9

Solution to Exercise 8.3.27 (p. 487)

3

50

Solution to Exercise 8.3.29 (p. 487)

4, 900 + 2, 700 = 7, 600 (7, 586)

Solution to Exercise 8.4.1 (p. 489)

6 · 8 + 6 · 4 = 48 + 24 = 72

Solution to Exercise 8.4.2 (p. 489)

4 · 4 + 4 · 7 = 16 + 28 = 44

Solution to Exercise 8.4.3 (p. 489)

8 · 2 + 8 · 9 = 16 + 72 = 88

Solution to Exercise 8.4.4 (p. 489)

12 · 10 + 12 · 3 = 120 + 36 = 156

Solution to Exercise 8.4.5 (p. 489)

6 · 11 − 6 · 3 = 66 − 18 = 48

Solution to Exercise 8.4.6 (p. 489)

8 · 9 − −8 · 7 = 72 − −56 = 16

Solution to Exercise 8.4.7 (p. 489)

15 · 30 − 15 · 8 = 450 − 120 = 330

Solution to Exercise 8.4.8 (p. 491)

25 (10 + 2) = 25 · 10 + 25 · 2 = 250 + 50 = 300

Solution to Exercise 8.4.9 (p. 491)

35 (10 + 4) = 35 · 10 + 35 · 4 = 350 + 140 = 490

Solution to Exercise 8.4.10 (p. 491)

80 (50 + 8) = 80 · 50 + 80 · 8 = 4, 000 + 640 = 4, 640

Solution to Exercise 8.4.11 (p. 491)

65 (60 + 2) = 65 · 60 + 65 · 2 = 3, 900 + 130 = 4, 030

Solution to Exercise 8.4.12 (p. 491)

15 (10 + 3) = 150 + 45 = 195

Solution to Exercise 8.4.14 (p. 491)

25 (10 + 1) = 250 + 25 = 275

Solution to Exercise 8.4.16 (p. 491)

15 (20 − 4) = 300 − 60 = 240

Solution to Exercise 8.4.18 (p. 491)

45 (80 + 3) = 3600 + 135 = 3735

Solution to Exercise 8.4.20 (p. 491)

25 (40 − 2) = 1, 000 − 50 = 950

Available for free at Connexions <http://cnx.org/content/col10615/1.4>

508

CHAPTER 8. TECHNIQUES OF ESTIMATION

Solution to Exercise 8.4.22 (p. 492)

75 (10 + 4) = 750 + 300 = 1, 050

Solution to Exercise 8.4.24 (p. 492)

65 (20 + 6) = 1, 300 + 390 = 1, 690 or 65 (30 − 4) = 1, 950 − 260 = 1, 690

Solution to Exercise 8.4.26 (p. 492)

15 (100 + 7) = 1, 500 + 105 = 1, 605

Solution to Exercise 8.4.28 (p. 492)

35 (400 + 2) = 14, 000 + 70 = 14, 070

Solution to Exercise 8.4.30 (p. 492)

95 (10 + 2) = 950 + 190 = 1, 140

Solution to Exercise 8.4.32 (p. 492)

80 (30 + 2) = 2, 400 + 160 = 2, 560

Solution to Exercise 8.4.34 (p. 492)

50 (60 + 3) = 3, 000 + 150 = 3, 150

Solution to Exercise 8.4.36 (p. 492)

40 (90 − 1) = 3, 600 − 40 = 3, 560

Solution to Exercise 8.4.38 (p. 492)

3

26

Solution to Exercise 8.4.40 (p. 492)

x = 42

Solution to Exercise 8.5.1 (p. 493)

Results may vary. 1 + 1 = 1. In fact, 5 + 5

2

2

8

12 = 25

24 = 1 1

24

Solution to Exercise 8.5.2 (p. 493)

Results may vary. 1 + 1 = 11. In fact, 7 + 3 = 117

2

2

9

5

45

Solution to Exercise 8.5.3 (p. 493)

Results may vary. 81 + 33 = 11 + 1 = 12. In fact, 8 4

4

4

15 + 3 7

10 = 11 29

30

Solution to Exercise 8.5.4 (p. 493)

Results may vary. (16 + 0) + (4 + 1) = 16 + 5 = 21. In fact, 16 120 + 47 = 2037

8

40

Solution to Exercise 8.5.5 (p. 494)

1 + 1 = 2

1 17

24

Solution to Exercise 8.5.7 (p. 494)

1 + 1 = 1 1 1 1

2

2

2

Solution to Exercise 8.5.9 (p. 494)

1 + 1 = 1 39

4

4

2

100

Solution to Exercise 8.5.11 (p. 494)

1 + 0 = 1 1 1

48

Solution to Exercise 8.5.13 (p. 494)

1 + 6 1 = 7 6 103

2

2

132

Solution to Exercise 8.5.15 (p. 494)

1 + 2 1 = 3 1 3 11

2

2

40

Solution to Exercise 8.5.17 (p. 494)

8 1 + 4 = 12 1 12 13

2

2

20

Solution to Exercise 8.5.19 (p. 494)

9 + 7 = 16

1513

15

Solution to Exercise 8.5.21 (p. 494)

3 1 + 2 1 + 2 = 8

7 189

2

2

200

Solution to Exercise 8.5.23 (p. 494)

1 − 1 = 0

1

16

Solution to Exercise 8.5.25 (p. 495)

associative

Available for free at Connexions <http://cnx.org/content/col10615/1.4>

509

Solution to Exercise 8.5.27 (p. 495)

8

9

Solution to Exercise 8.5.29 (p. 495)

25 (40 − 3) = 1000 − 75 = 925

Solution to Exercise 8.7.1 (p. 496)

600 (598)

Solution to Exercise 8.7.3 (p. 497)

(1,200)

Solution to Exercise 8.7.5 (p. 497)

4,900 (4,944)

Solution to Exercise 8.7.7 (p. 497)

3,800 (3,760)

Solution to Exercise 8.7.9 (p. 497)

6,050 (6,017)

Solution to Exercise 8.7.11 (p. 497)

6,700 (6,718)

Solution to Exercise 8.7.13 (p. 497)

61,400 (61,337)

Solution to Exercise 8.7.15 (p. 497)

89,700 (89,718)

Solution to Exercise 8.7.17 (p. 497)

213,000 (213,022)

Solution to Exercise 8.7.19 (p. 497)

3,900 (3,935)

Solution to Exercise 8.7.21 (p. 497)

2,400 (2,583)

Solution to Exercise 8.7.23 (p. 497)

600 (504)

Solution to Exercise 8.7.25 (p. 497)

150,123 147,000 (150,123)

Solution to Exercise 8.7.27 (p. 498)

47,278 48,000 (47,278)

Solution to Exercise 8.7.29 (p. 498)

12,771 14,100 (12,711)

Solution to Exercise 8.7.31 (p. 498)

8,100 (9,768)

Solution to Exercise 8.7.33 (p. 498)

90,000 (86,784)

Solution to Exercise 8.7.35 (p. 498)

396,000 (386,496)

Solution to Exercise 8.7.37 (p. 498)

1,609,048 1,560,000 (1,609,048)

Solution to Exercise 8.7.39 (p. 498)

22,050,000 (22,164,232)

Solution to Exercise 8.7.41 (p. 498)

18 (15)

Solution to Exercise 8.7.43 (p. 498)

141 (13)

2

Solution to Exercise 8.7.45 (p. 498)

50 (53)

Available for free at Connexions <http://cnx.org/content/col10615/1.4>

510

CHAPTER 8. TECHNIQUES OF ESTIMATION

Solution to Exercise 8.7.47 (p. 498)

54 (51)

Solution to Exercise 8.7.49 (p. 499)

130 (105)

Solution to Exercise 8.7.51 (p. 499)

41.25 (44)

Solution to Exercise 8.7.53 (p. 499)

42 (43)

Solution to Exercise 8.7.55 (p. 499)

14.4 (14)

Solution to Exercise 8.7.57 (p. 499)

60 (61)

Solution to Exercise 8.7.59 (p. 499)

1,000 (992)

Solution to Exercise 8.7.61 (p. 499)

71.1 (71.17)

Solution to Exercise 8.7.63 (p. 499)

119.4 (119.31)

Solution to Exercise 8.7.65 (p. 499)

272 (271.8)

Solution to Exercise 8.7.67 (p. 499)

2,001 (2,020.952)

Solution to Exercise 8.7.69 (p. 499)

4.16 (3.9596)

Solution to Exercise 8.7.71 (p. 500)

2 (40) + 2 (50) = 180 (178)

Solution to Exercise 8.7.73 (p. 500)

2 (30) + 2 (60) = 180 (179)

Solution to Exercise 8.7.75 (p. 500)

2 (80) + 2 (50) = 260 (263)

Solution to Exercise 8.7.77 (p. 500)

3 (20) + 1 (10) = 70 (79)

Solution to Exercise 8.7.79 (p. 500)

2 (15) + 2 (80) = 190 (201)

Solution to Exercise 8.7.81 (p. 500)

3 (20) + 2 (30) + 40 = 160 (160)

Solution to Exercise 8.7.83 (p. 500)

2 (60) + 2 (20) + 2 (30) = 220 (219)

Solution to Exercise 8.7.85 (p. 500)

2 (90) + 20 = 200 (200)

Solution to Exercise 8.7.87 (p. 500)

3 (20) + 2 (40) = 140 (140)

Solution to Exercise 8.7.89 (p. 500)

3 (90) + 2 (30) = 330 (321)

Solution to Exercise 8.7.91 (p. 501)

15 (30 + 3) = 450 + 45 = 495

Solution to Exercise 8.7.93 (p. 501)

35 (40 − 4) = 1400 − 140 = 1, 260

Solution to Exercise 8.7.95 (p. 501)

85 (20 + 3) = 1, 700 + 225 = 1, 955

Available for free at Connexions <http://cnx.org/content/col10615/1.4>

511

Solution to Exercise 8.7.97 (p. 501)

30 (10 + 4) = 300 + 120 = 420

Solution to Exercise 8.7.99 (p. 501)

75 (20 + 3) = 1, 500 + 225 = 1, 725

Solution to Exercise 8.7.101 (p. 501)

15 (20 − 3) = 300 − 45 = 255

Solution to Exercise 8.7.103 (p. 501)

65 (10 + 4) = 650 + 260 = 910

Solution to Exercise 8.7.105 (p. 501)

60 (40 + 2) = 2, 400 + 120 = 2, 520

Solution to Exercise 8.7.107 (p. 501)

15 (100 + 5) = 1, 500 + 75 = 1, 575

Solution to Exercise 8.7.109 (p. 501)

45 (300 + 6) = 13, 500 + 270 = 13, 770

Solution to Exercise 8.7.111 (p. 502)

1 + 1 = 1 1 1 5

2

2

24

Solution to Exercise 8.7.113 (p. 502)

1 + 1 = 1

27

2

2

30 or 9

10

Solution to Exercise 8.7.115 (p. 502)

1 + 1 = 3 113

2

4

4

150

Solution to Exercise 8.7.117 (p. 502)

1 + 1 = 1 1 39

4

4

32 or 1 7

32

Solution to Exercise 8.7.119 (p. 502)

2 3 + 6 1 = 9 1 9 7

4

2

4

20

Solution to Exercise 8.7.121 (p. 502)

111 + 71 = 183 1823

4

2

4

30

Solution to Exercise 8.7.123 (p. 502)

6 + 2 + 8 1 = 16 1 16 11

4

4

30

Solution to Exercise 8.7.125 (p. 502)

101 + 7 + 81 = 253 2527

2

4

4

40

Solution to Exercise 8.8.1 (p. 502)

10,500 (10,505)

Solution to Exercise 8.8.2 (p. 502)

18,000 (18,038)

Solution to Exercise 8.8.3 (p. 503)

5,000 (5,011)

Solution to Exercise 8.8.4 (p. 503)

28,000 (27,406)

Solution to Exercise 8.8.5 (p. 503)

3,200 (3,116)

Solution to Exercise 8.8.6 (p. 503)

4,500 (4,692)

Solution to Exercise 8.8.7 (p. 503)

300,000 (288,576)

Solution to Exercise 8.8.8 (p. 503)

12 (14)

Solution to Exercise 8.8.9 (p. 503)

90 (95)

Solution to Exercise 8.8.10 (p. 503)

81 (81.00)

Available for free at Connexions <http://cnx.org/content/col10615/1.4>

512

CHAPTER 8. TECHNIQUES OF ESTIMATION

Solution to Exercise 8.8.11 (p. 503)

505 (505.0065)

Solution to Exercise 8.8.12 (p. 503)

4,371 (4,380.16)

Solution to Exercise 8.8.13 (p. 503)

2 (60) + 2 (90) = 300 (298)

Solution to Exercise 8.8.14 (p. 503)

2 (40) + 2 (90) = 260 (263)

Solution to Exercise 8.8.15 (p. 503)

30 + 3 (80) = 270 (265)

Solution to Exercise 8.8.16 (p. 503)

3 (800) + 2 (600) = 3, 600 (3,618)

Solution to Exercise 8.8.17 (p. 503)

25 (10 + 4) = 250 + 100 = 350

Solution to Exercise 8.8.18 (p. 503)

15 (80 + 3) = 1, 200 + 45 = 1, 245

Solution to Exercise 8.8.19 (p. 503)

65 (100 − 2) = 6, 500 − 130 = 6, 370

Solution to Exercise 8.8.20 (p. 503)

80 (100 + 7) = 8, 000 + 560 = 8, 560

Solution to Exercise 8.8.21 (p. 503)

400 (200 + 15) = 80, 000 + 6, 000 = 86, 000

Solution to Exercise 8.8.22 (p. 503)

1 + 1 = 1 1 1 9

2

2

16

Solution to Exercise 8.8.23 (p. 503)

0 + 1 + 1 = 1 1 47

2

2

300

Solution to Exercise 8.8.24 (p. 504)

8 1 + 14 = 22 1 22 31

2

2

48

Solution to Exercise 8.8.25 (p. 504)

5 1 + 1 1 + 6 1 = 13 1 13 1

2

2

2

2

3

Available for free at Connexions <http://cnx.org/content/col10615/1.4>

Chapter 9

Measurement and Geometry

9.1 Objectives1

After completing this chapter, you should

Measurement and the United States System (Section 9.2)

• know what the word measurement means

• be familiar with United States system of measurement

• be able to convert from one unit of measure in the United States system to another unit of measure The Metric System of Measurement (Section 9.3)

• be more familiar with some of the advantages of the base ten number system

• know the prexes of the metric measures

• be familiar with the metric system of measurement

• be able to convert from one unit of measure in the metric system to another unit of measure Simplication of Denominate Numbers (Section 9.4)

• be able to convert an unsimplied unit of measure to a simplied unit of measure

• be able to add and subtract denominate numbers

• be able to multiply and divide a denominate number by a whole number

Perimeter and Circumference of Geometric Figures (Section 9.5)

• know what a polygon is

• know what perimeter is and how to nd it

• know what the circumference, diameter, and radius of a circle is and how to nd each one

• know the meaning of the symbol π and its approximating value

• know what a formula is and four versions of the circumference formula of a circle Area and Volume of Geometric Figures and Objects (Section 9.6)

• know the meaning and notation for area

• know the area formulas for some common geometric gures

• be able to nd the areas of some common geometric gures

• know the meaning and notation for volume

• know the volume formulas for some common geometric objects

• be able to nd the volume of some common geometric objects

1This content is available online at <http://cnx.org/content/m18897/1.3/>.

Available for free at Connexions <http://cnx.org/content/col10615/1.4> 513

514

CHAPTER 9. MEASUREMENT AND GEOMETRY

9.2 Measurement and the United States System2

9.2.1 Section Overview

• Measurement

• The United States System of Measurement

• Conversions in the United States System

9.2.2 Measurement

There are two major systems of measurement in use today. They are the United States system and the metric system. Before we describe these systems, let’s gain a clear understanding of the concept of measurement.

Measurement

Measurement is comparison to some standard.

Standard Unit of Measure

The concept of measurement is based on the idea of direct comparison. This means that measurement is the result of the comparison of two quantities. The quantity that is used for comparison is called the standard unit of measure.

Over the years, standards have changed. Quite some time in the past, the standard unit of measure was determined by a king. For example,

1 inch was the distance between the tip of the thumb and the knuckle of the king.

1 inch was also the length of 16 barley grains placed end to end.

Today, standard units of measure rarely change. Standard units of measure are the responsibility of the Bureau of Standards in Washington D.C.

Some desirable properties of a standard are the following:

1. Accessibility. We should have access to the standard so we can make comparisons.

2. Invariance. We should be condent that the standard is not subject to change.

3. Reproducibility. We should be able to reproduce the standard so that measurements are convenient and accessible to many people.

9.2.3 The United States System of Measurement

Some of the common units (along with their abbreviations) for the United States system of measurement are listed in the following table.

Unit Conversion Table

Length

1 foot (ft) = 12 inches (in.)

1 yard (yd) = 3 feet (ft)

1 mile (mi) = 5,280 feet

continued on next page

2This content is available online at <http://cnx.org/content/m35018/1.2/>.

Available for free at Connexions <http://cnx.org/content/col10615/1.4>

515

Weight

1 pound (lb) =16 ounces (oz)

1 ton (T) = 2,000 pounds

Liquid Volume

1 tablespoon (tbsp) = 3 teaspoons (tsp)

1 uid ounce ( oz) = 2 tablespoons

1 cup (c) = 8 uid ounces

1 pint (pt) = 2 cups

1 quart (qt) = 2 pints

1 gallon (gal) = 4 quarts

Time

1 minute (min) = 60 seconds (sec)

1 hour ( hr) = 60 minutes

1 day (da) = 24 hours

1 week (wk) = 7 days

Table 9.1

9.2.4 Conversions in the United States System

It is often convenient or necessary to convert from one unit of measure to another. For example, it may be convenient to convert a measurement of length that is given in feet to one that is given in inches. Such conversions can be made using unit fractions.

Unit Fraction

A unit fraction is a fraction with a value of 1.

Unit fractions are formed by using two equal measurements. One measurement is placed in the numerator of the fraction, and the other in the denominator. Placement depends on the desired conversion.

Placement of Units

Place the unit being converted to in the numerator.

Place the unit being converted from in the denominator.

For example,

Equal Measurements Unit Fraction

1ft = 12in.

1ft

12in. or 12in.

1ft

1pt = 16 oz

1pt

16 oz or 16 oz

1pt

1wk = 7da

7da

1wk or 1wk

7da

Table 9.2

9.2.4.1 Sample Set A

Make the following conversions. If a fraction occurs, convert it to a decimal rounded to two decimal places.

Example 9.1

Convert 11 yards to feet.

Looking in the unit conversion table under length, we see that 1 yd = 3 ft. There are two corresponding unit fractions, 1 yd

3 ft and 3 ft

1 yd . Which one should we use? Look to see which unit we wish

to convert to. Choose the unit fraction with this unit in the numerator. We will choose 3 ft 1 yd since

Available for free at Connexions <http://cnx.org/content/col10615/1.4>

516

CHAPTER 9. MEASUREMENT AND GEOMETRY

this unit fraction has feet in the numerator. Now, multiply 11 yd by the unit fraction. Notice that since the unit fraction has the value of 1, multiplying by it does not change the value of 11 yd.

11 yd =

11yd · 3ft

1

1yd

Divide out common units.

=

11)yd · 3ft

(Units can be added, subtracted, multiplied, and divided, just as numbers can.) 1

1)yd

=

11·3ft

1

=

33ft

Thus, 11 yd = 33ft.

Example 9.2

Convert 36 oz to pints.

Looking in the unit conversion table under liquid volume, we see that 1 pt = 16 oz. Since we are to convert to pints, we will construct a unit fraction with pints in the numerator.

36 oz =

36 oz · 1 pt

1

16 oz

Divide out common units.

=

36) oz · 1 pt

1

16) oz

=

36·1 pt

16

=

36 pt

Reduce.

16

=

9 pt

Convert to decimals: 9 = 2.25.

4

4

Thus, 36 oz = 2.25 pt.

Example 9.3

Convert 2,016 hr to weeks.

Looking in the unit conversion table under time, we see that 1wk = 7da and that 1 da = 24 hr.

To convert from hours to weeks, we must rst convert from hours to days and then from days to weeks. We need two unit fractions.

The unit fraction needed for converting from hours to days is 1 da

24 hr . The unit fraction needed for

converting from days to weeks is 1 wk

7 da .

2, 016 hr =

2,016 hr · 1 da

1

24 hr · 1 wk

7 da

Divide out common units.

=

2,016) hr · 1) da · 1 wk

1

24) hr

7) da

=

2,016·1wk

Reduce.

24·7

=

12wk

Thus, 2,016 hr = 12 wk.

9.2.4.2 Practice Set A

Make the following conversions. If a fraction occurs, convert it to a decimal rounded to two decimal places.

Exercise 9.2.1

(Solution on p. 569.)

Convert 18 ft to yards.

Available for free at Connexions <http://cnx.org/content/col10615/1.4>

517

Exercise 9.2.2

(Solution on p. 569.)

Convert 2 mi to feet.

Exercise 9.2.3

(Solution on p. 569.)

Convert 26 ft to yards.

Exercise 9.2.4

(Solution on p. 569.)

Convert 9 qt to pints.

Exercise 9.2.5

(Solution on p. 569.)

Convert 52 min to hours.

Exercise 9.2.6

(Solution on p. 569.)

Convert 412 hr to weeks.

9.2.5 Exercises

Make each conversion using unit fractions. If fractions occur, convert them to decimals rounded to two decimal places.

Exercise 9.2.7

(Solution on p. 569.)

14 yd to feet

Exercise 9.2.8

3 mi to yards

Exercise 9.2.9

(Solution on p. 569.)

8 mi to inches

Exercise 9.2.10

2 mi to inches

Exercise 9.2.11

(Solution on p. 569.)

18 in. to feet

Exercise 9.2.12

84 in. to yards

Exercise 9.2.13

(Solution on p. 569.)

5 in. to yards

Exercise 9.2.14

106 ft to miles

Exercise 9.2.15

(Solution on p. 569.)

62 in. to miles

Exercise 9.2.16

0.4 in. to yards

Exercise 9.2.17

(Solution on p. 569.)

3 qt to pints

Exercise 9.2.18

5 lb to ounces

Exercise 9.2.19

(Solution on p. 569.)

6 T to ounces

Exercise 9.2.20

4 oz to pounds

Exercise 9.2.21

(Solution on p. 569.)

15,000 oz to pounds

Available for free at Connexions <http://cnx.org/content/col10615/1.4>

518

CHAPTER 9. MEASUREMENT AND GEOMETRY

Exercise 9.2.22

15,000 oz to tons

Exercise 9.2.23

(Solution on p. 569.)

9 tbsp to teaspoons

Exercise 9.2.24

3 c to tablespoons

Exercise 9.2.25

(Solution on p. 569.)

5 pt to uid ounces

Exercise 9.2.26

16 tsp to cups

Exercise 9.2.27

(Solution on p. 569.)

5 oz to quarts

Exercise 9.2.28

3 qt to gallons

Exercise 9.2.29

(Solution on p. 569.)

5 pt to teaspoons

Exercise 9.2.30

3 qt to tablespoons

Exercise 9.2.31

(Solution on p. 569.)

18 min to seconds

Exercise 9.2.32

4 days to hours

Exercise 9.2.33

(Solution on p. 569.)

3 hr to days

Exercise 9.2.34

1 hr to days

2

Exercise 9.2.35

(Solution on p. 569.)

1 da to weeks

2

Exercise 9.2.36

3 1 wk to seconds

7

9.2.5.1 Exercises for Review

Exercise 9.2.37

(Solution on p. 569.)

(Section 2.5) Specify the digits by which 23,840 is divisible.

Exercise 9.2.38

(Section 4.5) Find 24 of 55 of 75.

5

6

7

Exercise 9.2.39

(Solution on p. 569.)

() Convert 0.32 to a fraction.

3

Exercise 9.2.40

(Section 8.3) Use the clustering method to estimate the sum: 53 + 82 + 79 + 49.

Exercise 9.2.41

(Solution on p. 569.)

(Section 8.4) Use the distributive property to compute the product: 60 · 46.

Available for free at Connexions <http://cnx.org/content/col10615/1.4>

519

9.3 The Metric System of Measurement3

9.3.1 Section Overview

• The Advantages of the Base Ten Number System

• Prexes

• Conversion from One Unit to Another Unit

• Conversion Table

9.3.2 The Advantages of the Base Ten Number System

The metric system of measurement takes advantage of our base ten number system. The advantage of the metric system over the United States system is that in the metric system it is possible to convert from one unit of measure to another simply by multiplying or dividing the given number by a power of 10. This means we can make a conversion simply by moving the decimal point to the right or the left.

9.3.3 Prexes

Common units of measure in the metric system are the meter (for length), the liter (for volume), and the gram (for mass). To each of the units can be attached a prex. The metric prexes along with their meaning are listed below.

Metric Prexes

kilo thousand

deci tenth

hecto hundred

centi hundredth

deka ten

milli thousandth

For example, if length is being measured,

1 kilometer is equivalent to 1000 meters.

1 centimeter is equivalent to one hundredth of a meter.

1 millimeter is equivalent to one thousandth of a meter.

9.3.4 Conversion from One Unit to Another Unit

Let’s note three characteristics of the metric system that occur in the metric table of measurements.

1. In each category, the prexes are the same.

2. We can move from a larger to a smaller unit of measure by moving the decimal point to the right.

3. We can move from a smaller to a larger unit of measure by moving the decimal point to the left.

The following table provides a summary of the relationship between the basic unit of measure (meter, gram, liter) and each prex, and how many places the decimal point is moved and in what direction.

kilo hecto deka unit deci centi milli

3This content is available online at <http://cnx.org/content/m35019/1.2/>.

Available for free at Connexions <http://cnx.org/content/col10615/1.4>

520

CHAPTER 9. MEASUREMENT AND GEOMETRY

Basic Unit to Prex

Move the Decimal Point

unit to deka

1 to 10

1 place to the left

unit to hector

1 to 100

2 places to the left

unit to kilo

1 to 1,000 3 places to the left

unit to deci

1 to 0.1

1 place to the right

unit to centi

1 to 0.01

2 places to the right

unit to milli

1 to 0.001 3 places to the right

Table 9.3

9.3.5 Conversion Table

Listed below, in the unit conversion table, are some of the common metric units of measure.

Unit Conversion Table

1

kilometer

(km)

=

1, 000 × 1 m

1,000 meters (m)

1 hectometer

(hm)

=

100 × 1 m

100 meters

Length

1 dekameter

(dam)

=

10 × 1m

10 meters

1 meter (m)

1 × 1 m

1 decimeter (dm) = 1

10 meter

.1 × 1 m

1 centimeter (cm) = 1

100 meter

.01 × 1 m

1 millimeter

(mm)

=

.001 × 1 m

1

1,000 meter

1 kilogram

(kg)

=

1, 000 × 1 g

1,000 grams (g)

1 hectogram (hg) = 100 grams

100 × 1 g

Mass

1 dekagram (dag) = 10 grams

10 × 1 g

1 gram (g)

1 × 1 g

1 decigram (dg) = 110 gram

.1 × 1 g

continued on next page

Available for free at Connexions <http://cnx.org/content/col10615/1.4>

Image 343

Image 344

521

1 centigram (cg) = 1

100 gram

.01 × 1 g

1 milligram

(mg)

=

.001 × 1 g

1

1,000 gram

1 kiloliter

(kL)

=

1, 000 × 1 L

1, 000 liters (L)

1 hectoliter (hL) = 100 liters

100 × 1 L

License

Math Copyright © by sboschman. All Rights Reserved.

Share This Book